You are on page 1of 28

MINISTRY OF PUBLIC HEALTH OF UKRAINE

Department of human resources policy, education and science

Testing Board

Student ID Last name

Variant ___________________

Test items for licensing examination

Krok 2
MEDICINE
General Instruction
Every one of these numbered questions or unfinished statements in
this chapter corresponds to answers or statements endings. Choose
the answer (finished statements) that fits best and fill in the circle
with the corresponding Latin letter on the answer sheet.

Authors of items: Abaturov O.Ye., Agarkov V.I., Alexeyenko L.I., Andrusha A.B., Artamonov V.S.,
Badogina L.P., Barannyk S.I., Belyakova V.I., Berezov V.M., Bezborod’ko S.A., Bilous T.M.,
Blikhar V.Ye., Bondarenko V.V., Borovkova S.O., Borysova T.P., Borzova O.Yu., Bukhteyeva E.R.,
Burka O.A., Buryak V.M., Butvyn I.M., Chonka I.I., Demchenko T.V., Derkach V.G., Detsyk O.Z.,
Dobrovols’ka L.M., Dzis’ N.P., Dzyuba G.A., Emiraliyeva Z.R., Franchuk O.A., Galushchenko S.A.,
Gendeleka G.F., Genyk N.I., Gerasymenko O.I., Goryachev V.V., Grebenyuk V.I., Grydasova V.D.,
Gurenko O.G., Gusak O.M., Ivanits’ka O.V., Ivanyuta S.O., Kaporina N.V., Karliychuk O.O.,
Khymenko M.F., Kol’tsova N.I., Kolyadenko N.V., Kondratenko P.G., Konopkina L.I., Koval’ova O.M.,
Kovtun A.I., Kovtunenko R.V., Kravchenko O.V., Krut’ Yu.Ya., Krylova V.Yu., Kryvonosov M.V.,
Kubyschkyn V.F., Kudiyevsky A.V., Kudrevych O.M., Kudrya V.I., Kuz’menko A.Ya., Kuz’menko S.A.,
Kvasnyts’ka O.B., Latypova G.A., Lishchenko N.O., Lysenko D.A., Lysenko V.M., Marichereda V.G.,
Martsynik Ye.M., Martynyuk L.P., Maruschko Yu.V., Mel’nychuk L.V., Mischchenko V.P., Moroz I.M.,
Myerkulova N.F., Mysak A.I., Nikitina N.O., Nitsovych R.M., Nishkumay O.I., Ognyev V.A.,
Okhrimenko G.I., Oliynyk O.Ye., Ostropolets’ M.S., Pavlovych L.V., Perepelyuk M.M., Perminova T.I.,
Pisots’ka L.A., Prokhoda A.F., Prokopchyk G.L., Prokopova L.V., Proniv L.M., Prunchak I.F., Prus L.O.,
Reva V.B., Romanenko V.N., Roshchupkina S.L., Rudenko M.M., Rudenko O.F., Ryapolova T.L.,
Rybalka A.M., Syabadash V.Ye., Sakevych P.P., Sannikova N.M., Selezn’ova S.V., Sels’ka O.V.,
Semenukha K.V., Semenyak A.V., Shcherbatyuk N.Yu., Shevchenko O.A., Shevchenko R.S., Shumko G.I.,
Shusterman T.Y., Sipityi V.I., Sichanova O.V., Slyepichko Yu.M., Slyva V.I., Smirnova V.L., Soldak I.I.,
Spirina I.D., Stepanyuk L.I., Sternyuk Yu.M., Svidro O.V., Svyrydova V.V., Syerkova V.K.,
Synovers’ka O.B., Sychova V.V., Tarallo V.L., Titarenko O.V., Tkachenko M.M., Tkachuk O.L.,
Tomash O.V., Tsyunchyk Yu.G., Tyuyeva N.V., Urbas’ O.V., Uryvayeva M.K., Usychenko O.M.,
Vankhanen N.V., Vankhanen V.D., Varbanets’ D.A., Vasil’yeva N.V., Vasyl’chyshyn Ya.V., Velyka N.V.,
Vereschagina N.Ya., Vinentsov Yu.O., Volyans’ka A.G., Vorkhlyk M.I., Vorokhta Yu.M.,
Yermachenko T.P., Yevdokimova T.B., Yevtushenko V.V., Yakovenko I.K., Yakubyshyna I.G.,
Yamilova T.M., Yudina T.V., Yurchenko I.V., Yurlov V.M., Yutanova A.V., Zakharchenko Yu.B.,
Zakharov V.K., Zavizion V.F., Zazdravnov A.A., Zelyak M.V., Zheliba M.D., Zhuravel V.I., Zhurilo I.P.,
Znakhurenko L.S., Zorya A.V., Zuban’ A.B. and Committees of professional expertise.

Item reviewers. Agafonova O.O., Anisimov Ye.M., Bab’yak T.Ye., Chursina T.Ya., Dyndar O.A.,
Grygorov Yu.B., Grynzovs’ky A.M., Gubka V.O., Gutsalenko O.O., Kalinina S.Yu., Karapetyan K.G.,
Khrapach V.V., Kolesnyk O.M., Kolosovych I.V., Kondratyuk V.Ye., Kopcha V.S., Kravchenko O.V.,
Kutovy O.B., Kuz’mina I.Yu., Malanchuk L.M., Martynyuk L.P., Mishchenko V.P., Muravs’ka O.M.,
Petrushenko V.V., Prokhorova M.P., Pryshlyak O.Ya., Puzanova O.G., Pyptyuk O.V., Shestakova I.V.,
Shevtsova T.I., Stovban I.V., Tsvirenko S.M., Usenko S.G., Vakal’uk I.P., Volyans’ka A.G.

The book includes test items for use at licensing integrated examination “Krok 2. Medicine” and
further use in teaching.

The book has been developed for students of medical, pediatric and medical-and-prophylactic
faculties and academic staff of higher medical educational establishments.

Approved by Ministry of Public Health of Ukraine as examination and teaching


publication based on expert conclusions (Orders of MPH of Ukraine of
14.08.1998 №251, of 27.12.1999 №303, of 16.10.2002 №374, of 29.05.2003 №233).

© Copyright Testing Board.


Крок 2 Medicine (англомовний варiант, iноземнi студенти) 2015 рiк 1

1. 13 months after the first labor a 24- A. Droperidolum of 0,25% - 2,0 ml


year-old patient complained of amenorrhea. B. Dibazolum (Bendazole hydrochloride) of
Pregnancy ended in Caesarian section because 1% - 6,0 ml
of premature detachment of normally positi- C. Papaverine hydrochloride of 2% - 4,0 ml
oned placenta which resulted in blood loss at D. Hexenalum of 1% - 2,0 ml
the rate of 2000 ml due to disturbance of blood E. Pentaminum of 5% - 4,0 ml
clotting. Choose the most suitable investigati-
on: 5. A patient with high temperature came to
a first-aid post in the evening. The fact of
A. Estimation of gonadotropin rate temporary disability was established. Indicate
B. US of small pelvis the measure to be taken in this case:
C. Progesteron assay
D. Computer tomography of head A. The night duty doctor should issue a medical
E. Estimation of testosteron rate in blood serum certificate, which will be subsequently used for
issuing a sick list from the date of the previous
2. A 24-year-old primipara was hospitalised day
with complaints of discharge of the amniotic B. The sick list for 1 day should be issued
waters. The uterus is tonic on palpation. The C. The sick list for up to 3 days should be issued
position of the fetus is longitudinal, it is pressed D. The sick list for 3 days should be issued
with the head to pelvic outlet. Palpitation of E. No document should be issued
the fetus is rhythmical, 140 bpm, auscultated
on the left below the navel. Internal examinati- 6. A 10-year-old boy complains of pain in his
on: cervix of the uterus is 2,5 cm long, dense, left eye and strong photophobia after he has
the external opening is closed, light amniotic injured his left eye with a pencil at school. Left
waters are discharged. Point out the correct eye examination revealed: blepharospasm, ci-
component of the diagnosis: liary and conjunctival congestion, cornea is
transparent, other parts of eyeball have no
A. Antenatal discharge of the amniotic waters changes. Visus 0,9. Right eye is healthy, Visus
B. Early discharge of the amniotic waters 1,0. What additional method would you choose
C. The beginning of the 1st stage of labour first of all?
D. The end of the 1st stage of labour
E. Pathological preterm labour A. Staining test with 1% fluorescein
B. X-ray examination of orbit
3. A patient was delivered to a surgical C. Tonometria
department after a road accident with a closed D. Gonioscopia
trauma of chest and right-sided rib fracture. E. Cornea sensation-test
The patient was diagnosed with right-sided
pneumothorax; it is indicated to perform drai- 7. An infant was born with body mass 3 kg and
nage of pleural cavity. Pleural puncture should body length 50 cm. Now he is 3 years old. His
be made: brother is 7 years old, suffers from rheumatic
fever. Mother asked a doctor for a cardiac
A. In the 2nd intercostal space along the middle check up of the 3-year-old son. Where is the
clavicular line left relative heart border located?
B. In the 6th intercostal space along the posteri-
or axillary line A. 1 cm left from the left medioclavicular line
C. In the 7th intercostal space along the scapular B. 1 cm right from the left medioclavicular line
line C. Along the left medioclavicular line
D. In the projection of pleural sinus D. 1 cm left from the left parasternal line
E. In the point of the greatest dullness on E. 1 cm right from the left parasternal line
percussion 8. A baby was born at 36 weeks of gestation.
4. A primagravida with pregnancy of 37-38 Delivery was normal, by natural way. The baby
weeks complains of headache, nausea, pain in has a large cephalohematoma. The results of
epigastrium. Objectively: the skin is acyanotic. blood count are: Hb- 120g/l, Er- 3, 5 · 1012 /l,
Face is hydropic, there is short fibrillar twi- total serum bilirubin - 123 mmol/l, direct bili-
tching of blepharons, muscles of the face and rubin - 11 mmol/l, indirect - 112 mmol/l. What
the inferior extremities. The stare is fixed. BP are the causes of hyperbilirubinemia in this
- 200/110 mm Hg; sphygmus is of 92 bpm, case?
intense. Respiration rate is 32/min. Heart acti-
vity is rhythmical. Appreciable edemas of A. Erythrocyte hemolysis
the inferior extremities are present. Urine is B. Intravascular hemolysis
cloudy. What medication should be admini- C. Disturbance of the conjugative function of
stered? liver
D. Bile condensing
E. Mechanical obstruction of the bile outflow
9. A patient has been hospitalised. The onset of
the disease was gradual: nausea, vomiting, dark
urine, аcholic stool, yellowness of the skin and
scleras. The liver is enlarged by 3 cm. Jaundice
Крок 2 Medicine (англомовний варiант, iноземнi студенти) 2015 рiк 2

developed on the 14th day of the disease. The 14. A 13-year-old girl complains of periodic
liver diminished in size. What complication of prickly pain in the heart region. Percussi-
viral hepatitis caused deterioration of the pati- on revealed no changes of cardiac borders.
ent’s condition? Auscultation revealed arrhythmic enhanced
heart sounds, extrasystole at the 20-25 cardi-
A. Hepatic encephlopathy ac impulse. ECG showed the sinus rhythm,
B. Meningitis impaired repolarization, single supraventri-
C. Relapse of viral hepatitis cular extrasystoles at rest. What is the most
D. Cholangitis likely diagnosis?
E. Infectious-toxic shock
A. Somatoform autonomic dysfunction
10. An 18-year-old patient was admitted B. Rheumatism
to a hospital with complaints of headache, C. Nonrheumatic carditis
weakness, high fever, sore throat. Objectively: D. Myocardial degeneration
enlargement of all groups of lymph nodes was E. Intoxication syndrome
revealed. The liver is enlarged by 3 cm, spleen
- by 1 cm. In blood: leukocytosis, atypical 15. A 24-year-old patient visited a doctor
lymphocytes - 15%. What is the most probable complaining of enlargement of his submaxi-
diagnosis? llary lymph nodes. Objectively: submaxillary,
axillary and inguinal lymph nodes are enlarged.
A. Infectious mononucleosis Chest X-ray shows: enlarged lymph nodes
B. Acute lymphoid leukosis of mediastinum. Blood test: erythrocytes -
C. Diphtheria 3, 4 · 1012 /l, Hb- 100 g/l, blood colour index
D. Angina
E. Adenoviral infection - 0,88, platelets - 190 · 109 /l, leucocytes -
7, 5 · 109 /l, eosinophiles - 8%, band neutrophi-
11. A 60-year-old woman, mother of 6 children, les - 2%, segmented neutrophiles - 67%,
developed a sudden onset of upper abdomi- lymphocytes - 23%, ESR - 22 mm/h. What
nal pain radiating to the back, accompani- test must be prescribed to verify the cause of
ed by nausea, vomiting, fever and chills. lymphadenopathy?
Subsequently, she noticed yellow discolorati-
on of her sclera and skin. On physical exami- A. Open biopsy of lymph nodes
nation the patient was found to be febrile with B. Ultrasonography of abdominal cavity
temperature of 38, 9o C , along with right upper C. Mediastinum tomography
quadrant tenderness. The most likely diagnosis D. Puncture biopsy of lymph nodes
is: E. Sternal puncture

A. Choledocholithiasis 16. A 60-year-old woman has been sufferi-


B. Benign biliary stricture ng from arterial hypertension for 15 years.
C. Malignant biliary stricture After recurrent stroke she started complaini-
D. Carcinoma of the head of the pancreas ng about unmotivated bad mood, problems
E. Choledochal cyst with attention concentration; she forgets to
close the entrance door, cannot recall events
12. What juice is recommended to be included of the past day. Computer tomography shows
in a complex drug and diet therapy for patients areas of postinfarction changes in the cortical
suffering from gastric or duodenal ulcer and postfrontal areas. What is the most probable
high acidity of gastric juice to speed up ulcer diagnosis?
healing?
A. Vascular dementia
A. Potato, potato-carrot B. Alzheimer’s disease
B. Apple, apple-birch C. Huntington’s disease
C. Pumpkin D. Pick’s disease
D. Cabbage, cabbage-carrot E. Dissociative amnesia
E. Celery, parsley
17. Clinic of a research instutute for occupati-
13. A 40-year-old woman has been hospitalized onal diseases examined a worker who works at
for attacks of asphyxia, cough with phlegm. She a concentration plant and diagnosed him with
has a 4-year history of the disease. The first chronic dust bronchitis. The case is investigated
attack of asphyxia occurred during her stay in by a commission including the representatives
the countryside. Further attacks occurred whi- of: the plant, medical unit, territorial sanitation
le cleaning the room. After 3 days of inpatient center, department of Social Insurance Fund,
treatment the patient’s condition has signifi- trade union. According to the "regulation on
cantly improved. What is the most likely eti- investigation of. . . ", the commission should be
ological factor? headed by the representative of the following
authority:
A. Household allergens
B. Pollen
C. Infectious
D. Chemicals
E. Psychogenic
Крок 2 Medicine (англомовний варiант, iноземнi студенти) 2015 рiк 3

A. Territorial sanitation center mes joined by hemoptysis and pain in the ri-
B. Plant ght side of chest. Breathing is vesicular. X-ray
C. Social Insurance Fund shows darkening and sharp decrease in size of
D. Trade union the lower lobe distinctly visible on the X-ray
E. Medical unit image as a streak 2-3 cm wide situated at the
angle from lung root to the frontal costodi-
18. 4 days after a patient received a gunshot aphragmatic recess. The most likely diagnosis
wound of the middle third of the thigh soft ti- is:
ssues his condition suddenly began deteriorati-
ng. There are complaints of bursting pain in the A. Peripheral lung cancer
wound; pain increases during the last 12 hours. B. Bronchiectasis
Edema of skin and hypodermic tissue quickly C. Pneumonia
grows. Body temperature is 38, 2o C , heart rate D. Middle lobe syndrome
is 102/min. The wound edges gape, are dull in E. Interlobular pleurisy
color; the muscles, viable as of day before, now
protrude into the wound, look boiled, are dull 23. A 52-year-old patient, who has been sufferi-
in colour, have dirty-grey coating and fall apart ng from angina pectoris for 2 weeks, has
when being held with forceps. What infection more and more frequent pain attacks in the
has developed in the wound? area behind his sternum and his need for ni-
troglycerine increased. Objectively: the condi-
A. Anaerobic tion is of moderate severity. Skin is pale. Heart
B. Aerobic gram-negative sounds are weakened, rhythmic. Heart rate is
C. Putrid 84 per minute. ECG shows no signs of focal
D. Aerobic gram-positive myocardial damage. What is the most likely di-
E. Diphtheria of wound agnosis?
19. Bacterial analysis of air in a living space in A. Progressive angina pectoris
winter period by means of Krotov’s apparatus B. First-time angina pectoris
revealed that total number of microorganisms C. Stable FC II angina pectoris
in 1m3 of air was 7200. What is the allowed D. Variant angina pectoris
number of microorganisms for the air to be E. Acute cardiac infarction
characterized as "pure"?
24. An 8-year-old boy during preventive exami-
A. Up to 4500 nation was determined to have changes in
B. Up to 2500 his spine curvature in frontal plane: the ri-
C. Up to 3500 ght shoulder is lowered and flat, scapulae
D. Up to 5500 angles are of different height due to the ri-
E. Up to 7500 ght scapula being shifted down. Waist triangles
are pronounced on the both sides; longitudinal
20. Maximum permissible concentration of muscles of the back form muscle cushion on
carbon dioxide in the air is considered to be the left. What type of posture is detected in the
a sanitary index of air purity in a classroom. child?
What concentration of carbon dioxide in the
air is accepted as a permissible maximum? A. Scoliotic
B. Kyphotic
A. 0,1% C. Lordotic
B. 0,05% D. Stooping
C. 0,15% E. Corrected
D. 0,2%
E. 0,3% 25. A patient with cardiac infarction is on sick
leave with his medical certificate being conti-
21. When examining the parameters of desk nuously extended for 4 months according to a
natural lighting in a school class it was determi- standard procedure. The disease is progressi-
ned that: light angle is 25o , window angle is ng. The issue is put forward to refer the pati-
3o , window-to-floor area ratio is 1:4, dayli- ent to the industrial injury assessment board.
ght factor is 0,5%, and window-head-to-room- Who among the medical staff of medical and
depth ratio is 2. What parameter does not preventive treatment facility is authorized to
correspond to hygienic norms? do this?
A. Daylight factor A. Medical expert committee
B. Window angle B. Attending medical doctor
C. Window-to-floor area ratio C. Head doctor of hospital department
D. Window-head-to-room-depth ratio D. Deputy head doctor for assessment of
E. Light angle temporary disability
E. Head doctor
22. A 29-year-old patient works as a motor
mechanic. Anamnesis shows frequent exposure 26. A 9-year-old boy has been suffering from
to cold, exacerbation of chronic bronchitis bronchoectasis since he was 3. Exacerbations
attended by cough with relativly small amount occur quite often, 3-4 times a year. Conservati-
of mucopurulent sputum, subfebrility, someti- ve therapy results in short periods of remi-
Крок 2 Medicine (англомовний варiант, iноземнi студенти) 2015 рiк 4

ssion. The disease is progressing, the child 31. A 46-year-old patient once took part in eli-
has physical retardation. The child’s skin is mination of breakdown at an atomic power
pale, acrocyanotic, he has "watch glass"nail plant. Currently he is being treated at an in-
deformation. Bronchography revealed saccular patient hospital unit. He was diagnosed with
bronchiectases of the lower lobe of his right progressing somatoform autonomic dysfuncti-
lung. What is the further treatment tactics? on. This disease relates to the following group
of ionizing radiation effects:
A. Surgical treatment
B. Further conservative therapy A. Somato-stochastic
C. Physiotherapeutic treatment B. Somatic
D. Sanatorium-and-spa treatment C. Genetic
E. Tempering of the child’s organism D. Hormesis
E. Heterosis
27. A 29-year-old female patient complains of
dyspnea, heaviness and chest pain on the ri- 32. A patient complains of fatigue, lack of
ght, body temperature rise up to 37, 2o C . The appetite, pain and burning sensation in the
disease is associated with a chest trauma recei- tongue, numbness of the distal limbs, diarrhea.
ved 4 days ago. Objectively: skin is pale and Objectively: pale skin with lemon-yellow tint,
moist. Heart rate is 90 bpm, regular. Palpation face puffiness, brown pigmentation in the form
reveals a dull sound on the right, auscultati- of a "butterfly", bright red areas on the tongue.
on reveals significantly weakened vesicular The liver is 3 cm below the costal margin, soft.
breathing. In blood: RBCs - 2, 8 · 1012 /l, colour Blood count: RBCs - 1, 5 · 1012 /l, colour index -
index - 0,9, Hb- 100 g/l, WBCs - 8, 0 · 109 /l, ESR 1,2, WBCs - 3, 8·109 /l, thrombocytes - 180·109 /l,
- 17 mm/h. What results of diagnostic puncture eosinophiles - 0%, stab neutrophiles - 1%,
of the pleural cavity can be expected? segmented neutrophiles - 58%, lymphocytes -
38% monocytes - 3%, RBC macrocytosis. ESR
A. Haemorrhagic punctate - 28 mm/h. What diagnosis are these presentati-
B. Chylous liquid ons typical for?
C. Exudate
D. Transudate A. B12 -deficiency anemia
E. Purulent punctate B. Iron deficiency anemia
C. Aplastic anemia
28. A 54-year-old man had been drowning at D. Acute erythromyelosis
sea, when he was found and evacuated to the E. Chronic adrenal failure
shore. Objectively: unconscious, pale face, no
breathing can be auscultated, thready pulse. 33. A 25-year-old female patient complains of
Resuscitation measures allowed to save the marked weakness, sleepiness, blackouts, di-
man. What complication can develop in him zziness, taste disorder. The patient has a hi-
in the nearest future? story of menorrhagia. Objectively: the pati-
ent has marked weakness, pale skin, cracks
A. Pulmonary edema in the corners of her mouth, peeling nails,
B. Respiratory arrest systolic apical murmur. Blood test results: RBC
C. Encephalopathy - 3, 4 · 1012 /l, Hb- 70 g/l, colour index - 0,75,
D. Cardiac arrest
E. Bronchial spasm platelets - 140 · 109 /l, WBC- 6, 2 · 109 /l. What is
the most likely diagnosis?
29. A 7-year-old child complains of cramping
pain occuring after mental exertion, cold dri- A. Chronic posthemorrhagic anemia
nks and eating ice-cream. Instrumental exami- B. Acute leukemia
nation allowed to diagnose biliary dyskinesia of C. Acute posthemorrhagic anemia
hypertensive type. What group of drugs should D. B12 -deficiency anemia
be prescribed for treatment? E. Werlhof’s disease

A. Antispasmodics and choleretics 34. A 39-year-old patient complains of morning


B. Choleretics and cholekinetics headache, appetite loss, nausea, morning vomi-
C. Sedatives and cholekinetics ting, periodic nasal haemorrhages. The patient
D. Antioxydants had acute glomerulonephritis at the age of 15.
E. Antibiotics Examination revealed rise of arterial pressure
up to 220/130 mm Hg, skin haemorrhages on
30. Caries morbidity rate is 89% among resi- his arms and legs, pallor of skin and mucous
dents of a community. It is determined that membranes. What biochemical parameter is
fluorine content in water is 0,1 mg/l. What the most important for making diagnosis in
preventive measures should be taken? this case?

A. Water fluorination A. Blood creatinine


B. Tooth brushing B. Blood bilirubin
C. Fluorine inhalations C. Blood sodium
D. Sealant application D. Uric acid
E. Introduce more vegetables to the diet E. Fibrinogen
Крок 2 Medicine (англомовний варiант, iноземнi студенти) 2015 рiк 5

35. A child was taken to a hospital with focal treatment is going to last for another 1-2
changes in the skin folds. The child was anxi- months. Who is authorised to extend the
ous during examination, examination revealed duration of medical certificate for this patient?
dry skin with solitary papulous elements and
ill-defined lichenification zones. Skin erupti- A. Medical advisory commission after medico-
on was accompanied by strong itch. The child social expert commission examination
usually feels better in summer, his condition B. Medical superintendent
is getting worse in winter. The child has been C. Medical advisory commission after inpatient
on bottle feeding since he was 2 months old. treatment
He has a history of exudative diathesis. His D. District doctor by agreement with a
grandmother on his mother’s side has bronchi- department chief
al asthma. What is the most likely diagnosis? E. Medico-social expert commission

A. Atopic dermatitis 40. A 30-year-old woman complains of pain


B. Contact dermatitis in the heart area ("aching, piercing pain")
C. Seborrheal eczema that arises primarily in the morning hours in
D. Strophulus autumn and spring. Pain irradiates into the
E. Urticaria neck, back, abdomen and is attended by rapid
heart rate and low vital tonus. This conditi-
36. An electro-gas welding operator worki- on occurs independently from physical exerti-
ng at a machine workshop performs welding on. In the evening her condition improves.
and cutting of metal, which is accompanied by Somatic and neurologic state and ECG have no
intense UV-radiation. His welding station is pathologies. What pathology is likely to result
equipped with effective mechanical ventilati- in such clinical presentation?
on. What occupational disease is most likely to
develop in the electro-gas welding operator? A. Somatized depression
B. Rest angina pectoris
A. Photoelectric ophthalmia C. Pseudoneurotic schizophrenia
B. Heatstroke D. Somatoform autonomic dysfunction
C. Vegetative-vascular dystonia E. Hypochondriacal depression
D. Chronic overheating
E. Pneumoconiosis 41. A 59-year-old female patient attended
a maternity welfare clinic with complains
37. 2 weeks after recovering from tonsillitis an of bloody discharge from the genital tracts.
8-year-old boy developed edemas of face and Postmenopause is 12 years. Vaginal exami-
lower limbs. Objectively: the patient is in grave nation revealed that external genital organs
condition, BP - 120/80 mm Hg. Urine is of dark had signs of age involution, uterus cervix
brown colour. Oliguria is present. On urine was not erosive, small amount of bloody di-
analysis: specific gravity - 1,015, protein - 1,2 scharge came from the cervical canal. Uterus
g/l, RBCs are leached and cover the whole vi- is of normal size, uterine appendages are
sion field, granular casts - 1-2 in the vision field, unpalpable. Fornices were deep and painless.
salts are represented by urates (large quantity). What method should be applied for the di-
What is the most likely diagnosis? agnosis specification?
A. Acute glomerulonephritis with nephritic A. Separated diagnosic curretage
syndrome B. Laparoscopy
B. Acute glomerulonephritis with nephrotic C. Puncture of abdominal cavity through
syndrome posterior vaginal fornix
C. Acute glomerulonephritis with nephrotic D. Extensive colposcopy
syndrome, hematuria and hypertension E. Culdoscopy
D. Acute glomerulonephritis with isolated
urinary syndrome 42. A 41-year-old patient, a hunter, complai-
E. Nephrolithiasis ns of heaviness in the right subcostal area.
No other complaints recorded. Anamnesis
38. A 48-year-old patient has the followi- states causeless urticaria and skin itching that
ng symptoms: diffuse enlargement of thyroid occurred 1 year ago. Objectively: liver margin
gland, exophthalmus, weight loss up to 4 kg wi- is rounded, painless, and can be palpated 3
thin 2 months, excessive sweating. Objectively: cm below the costal arch. No other pathology
heart rate 105/min, BP - 180/70 mm Hg. Stool is was detected during physical examination of
normal. What therapy is advisable in the given the patient. Body temperature is normal. X-ray
case? reveals a hemispherical protrusion in the right
cupula of the diaphragm. What disease can be
A. Mercazolil (Thiamazole) suspected in the given case?
B. Potassium iodide
C. Propranolol A. Hydatid disease of liver
D. Iodomarin B. Liver abscess
E. Thyroxin C. Hepatocellular carcinoma
D. Metastatic tumor
39. A patient is on sick leave for 4 months E. Subdiaphragmatic abscess
continuously from the date of injury. The
Крок 2 Medicine (англомовний варiант, iноземнi студенти) 2015 рiк 6

43. A 14-year-old girl has been presenting wi- Objectively: focal neurological symptoms
th irritability and tearfulness for about a year. were not found. Pronounced meningeal
A year ago she was also found to have diffuse symptoms were revealed. BP - 120/80 mm
enlargement of the thyroid gland (II grade). Hg. According to clinical and liquorologi-
This condition was regarded as a pubertal cal findings the patient was diagnosed wi-
manifestation, the girl did not undergo any th subarachnoid haemorrhage. After admi-
treatment. The girl’s irritability gradually gave nistration of dehydrants the patient’s condi-
place to a complete apathy. The girl got tion somewhat improved. What is the main
puffy face, soft tissues pastosity, bradycardia, component of further emergency care?
constipations. Skin pallor and gland density
progressed, the skin got a waxen hue. What di- A. Coagulants
sease may be assumed? B. Anticoagulants
C. Antiaggregants
A. Autoimmune thyroiditis D. Fibrinolytics
B. Diffuse toxic goiter E. Corticosteroids
C. Thyroid carcinoma
D. Subacute thyroiditis 48. A 42-year-old man has undergone gastric
E. Juvenile basophilism analysis. Free hydrochloric acid is absent
at all phases. On endoscopy: pallor, gastric
44. A 26-year-old woman, who delivered a child mucosa loss, gastric folds are smoothed out.
7 months ago, has been suffering from nausea, On microscopy: glands atrophy with intestinal
morning vomiting, sleepiness for the last 2 metaplasia. What disease can be characterized
weeks. She breastfeeds the child, menstruation by such presentation?
is absent. She has not applied any contracepti-
ves. What method should be applied in order A. Chronic gastritis type A
to specify her diagnosis? B. Chronic gastritis type B
C. Chronic gastritis type C
A. Ultrasonic examination D. Menetrier’s disease
B. X-ray of small pelvis E. Stomach cancer
C. Palpation of mammary glands and pressing-
out of colostrum 49. A 24-year-old woman, teacher by professi-
D. Bimanual vaginal examination on, complains of dizziness and heart pain
E. Speculum examination irradiating to the left nipple. Pain is not associ-
ated with physical activity and cannot be reli-
45. Over a current year among workers of an eved by nitroglycerin, it abates after taking
institution 10% have not been ill a single time, Valocordin and lasts for an hour or more. The
30% have been ill once, 15% - twice, 5% - 4 patient has a nearly 2-year history of this di-
times, the rest - 5 and more times. What is the sease. Objectively: Heart rate - 76 bpm. BP
percentage of workers belonging to the I health - 110/70 mm Hg. Heart borders are normal,
group? heart sounds are clear. The ECG shows respi-
ratory arrhythmia. X-ray of the cervicothoracic
A. 55% spine shows no pathology. Lungs, abdomen are
B. 10% unremarkable. What changes in blood formula
C. 40% can be expected?
D. 60%
E. 22% A. No changes
B. Leukocytosis
46. A 34-year-old patient after vacation in C. Thrombocytopenia
Crimea has developed severe pain in her D. Leukemic hiatus
elbow joints, dyspnea and weakness. Body E. Increased ESR
temperature is 37, 6o C , skin pallor and
erythema on the cheeks and bridge of nose 50. Chief physician of a polyclinic tasked a di-
are observed, lip mucosa is ulcerated. The joi- strict doctor with determining the pathologi-
nts are not visibly deformed, movement of the cal prevalence of disease N in his district.
right elbow joint is restricted. Pleura friction What document allows to estimate the disease
sound is detected in the lungs on the right side prevalence in the population of a medical di-
below the angle of scapula. Heart sounds are strict?
dull, tachycardia, gallop rhythm, heart rate -
114/min, BP - 100/60 mm Hg. The most likely A. Prophylactic examinations register
diagnosis is: B. Statistic coupons (+)
C. Statistic coupons (-)
A. Systemic lupus erythematosus D. Statistic coupons (+) and (-)
B. Rheumatic carditis E. Vouchers for medical appointments
C. Rheumatoid arthritis
D. Infectious allergic myocarditis 51. A 32-year-old woman complains of di-
E. Dry pleurisy zziness, headache, palpitation, tremor. For
the last several months she has been under
47. Several hours before, a 28-year-old pati- outpatient monitoring for increased arterial
ent suddenly developed acute headache and pressure. Recently such attacks have become
repeated vomiting, then lost consciousness. more frequent and severe. Objectively: skin
Крок 2 Medicine (англомовний варiант, iноземнi студенти) 2015 рiк 7

is covered with clammy sweat, tremor of the A. Coliphage number


extremities is present. Heart rate - 110/min, BP B. Colibacillus index
- 220/140 mm Hg. Heart sounds are weakened. C. Oxidability
In blood: WBCs - 9, 8 · 109 /l, ESR - 22 mm/h. D. Non-icteric leptospirosis agent
Blood glucose - 9,8 millimole/l. What disease is E. Fecal coliform bacteria index
the most likely cause of this crisis? 56. A 25-year-old patient was delivered to
A. Pheochromocytoma an infectious diseases unit on the 3rd day
B. Essential hypertension of illness with complaints of headache, pain
C. Preeclampsia in lumbar spine and gastrocnemius muscles,
D. Primary hyperaldosteronism high fever, chill. Objectively: condition of
E. Diabetic glomerulosclerosis moderate severity. Scleras are icteric. Pharynx
is hyperemic. Tongue is dry with dry brown
52. A 45-year-old patient complains of pain coating. Abdomen is distended. Liver is
in the epigastric region, left subcostal area, enlarged by 2 cm. Spleen is not enlarged.
abdominal distension, diarrhea, loss of wei- Palpation of muscles, especially gastrocnemi-
ght. He has been suffering from this condition us muscles, is painful. Urine is dark in colour.
for 5 years. Objectively: tongue is moist with Stool is normal in colour. The most likely di-
white coating near the root; deep palpation of agnosis is:
abdomen reveals slight pain in the epigastric
region and Мауо-Robson’s point. Liver is pai- A. Leptospirosis
nless and protrudes 1 cm from the costal arch. B. Viral hepatitis type A
Spleen cannot be palpated. What disease can C. Malaria
be primarily suspected? D. Infectious mononucleosis
E. Yersiniosis
A. Chronic pancreatitis
B. Atrophic gastritis 57. A patient suffering from acute
C. Peptic stomach ulcer posttraumatic pain received an injection of
D. Chronic cholecystitis morphine that brought him a significant relief.
E. Chronic enteritis Which of the following mechanisms of action
provided for antishock effect of morphine in
53. A 58-year-old woman complains of this patient?
spontaneous bruises, weakness, bleeding gums,
dizziness. Objectively: the mucous membranes A. Stimulation of opiate receptors
and skin are pale with numerous hemorrhages B. Block of central cholinergic receptors
of various time of origin. Lymph nodes are not C. Stimulation of benzodiazepine receptors
enlarged. Heart rate - 100/min, BP- 110/70 D. Inhibition of dopamine mediation
mm Hg. There are no changes of internal E. Intensification of GABA-ergic reactions
organs. Blood test results: RBC - 3,0·1012 /l, 58. A 9-month-old child presents with fever,
Нb - 92 g/l, colour index - 0,9, anisocytosis, poi- cough, dyspnea. The symptoms appeared 5
kilocytosis, WBC - 10·109 /l, eosinophiles - 2%, days ago after a contact with a person havi-
stab neutrophiles - 12%, segmented neutrophi- ng URTI. Objectively: the child is in grave
les - 68%, lymphocytes - 11%, monocytes - 7%, condition. Temperature of 38o C , cyanosis of
ESR - 12 mm/h. What laboratory test should be nasolabial triangle is present. Respiration rate
determined additionally to make a diagnosis? - 54/min, nasal flaring while breathing. There
was percussion dullness on the right below the
A. Platelets scapula angle, and tympanic sound over the
B. Reticulocytes rest of lungs. Auscultation revealed bilateral fi-
C. Clotting time ne moist rales (crackles) predominating on the
D. Osmotic resistance of erythrocytes right. What is the most likely diagnosis?
E. Fibrinogen
A. Acute pneumonia
54. After examination a 46-year-old patient was B. URTI
diagnosed with left breast cancer T2N2M0, cli- C. Acute laryngotracheitis
nical group II-a. What will be the treatment D. Acute bronchitis
plan for this patient? E. Acute bronchiolitis
A. Radiation therapy + operation + 59. A 50-year-old patient complains about havi-
chemotherapy ng pain attacks in the right subcostal area for
B. Operation only about a year. The pain arises mainly after
C. Operation + radiation therapy taking fatty food. Over the last week the
D. Radiation therapy only attacks occurred daily and became more pai-
E. Chemotherapy only nful. On the 3rd day of hospitalization the pati-
55. In river-side urban community there was an ent presented with icteritiousness of skin and
outbreak of hepatitis type A possibly spread by scleras, light-coloured feces and dark urine. In
water. What indexes of river water quality can blood: neutrophilic leukocytosis - 13, 1 · 109 /l,
confirm this theory? ESR - 28 mm/h. What is the most likely di-
agnosis?
Крок 2 Medicine (англомовний варiант, iноземнi студенти) 2015 рiк 8

A. Chronic calculous cholecystitis other day. Two days ago the patient arrived
B. Chronic recurrent pancreatitis from Africa. Objectively: pale skin, subicteric
C. Fatty degeneration of liver sclera, significantly enlarged liver and spleen.
D. Chronic cholangitis, exacerbation stage What is the cause of fever attacks in this di-
E. Hypertensive dyskinesia of gallbladder sease?
60. A 52-year-old woman complains of face di- A. Erythrocytic schizogony
stortion. It appeared 2 days ago after exposure B. Tissue schizogony
to cold. Objectively: body temperature is C. Exotoxin of a causative agent
38, 2o C . Face asymmetry is present. Frontal D. Endotoxin of a causative agent
folds are flattened. Left eye is wider than the ri- E. Gametocytes
ght one and cannot close. Left nasolabial fold is
flattened, mouth corner is lowered. Examinati- 65. A 49-year-old patient 2 years ago was di-
on revealed no other pathology. Blood count: agnosed with stage 1 silicosis. He complains of
leukocytes - 10 · 109 /l, ESR - 20 mm/h. What is increased dyspnea and pain in the infrascapular
the most probable diagnosis? regions. X-ray shows diffuse intensification
and distortion of lung pattern with numerous
A. Facial neuritis nodular shadows 2-4 mm in diameter. Hardeni-
B. Trigeminus neuralgia ng of right interlobar pleura is detected. Lung
C. Hemicrania (migraine) roots are hardened and expanded. What X-ray
D. Ischemic stroke type of pneumosclerosis does the patient have?
E. Brain tumour
A. Nodular
61. A 28-year-old female patient with a six-year B. Interstitial
history of Raynaud’s syndrome has recently C. Interstitial-nodular
developed pain in the small joints of hands, di- D. Nodal
fficult movement of food down the esophagus. E. Tumor-like
What disease can be suspected in this case?
66. A 24-year-old patient complains of general
A. Systemic scleroderma weakness, dizziness, body temperature ri-
B. Periarteritis nodosa se up to 37, 5o C , sore throat, neck edema,
C. Rheumatoid arthritis enlargement of submaxillary lymph nodes.
D. Systemic lupus erythematosus Objectively: mucous membrane of oropharynx
E. Pseudotrichiniasis is edematic and cyanotic, tonsils are enlarged
and covered with films that spread beyond
62. An 8-year-old boy, who has been sufferi- the tonsils and cannot be easily removed.
ng from diabetes mellitus for 3 years, was What is the leading mechanism of this illness
delivered to a hospital in a condition of development?
hyperglycemic coma. Primary dose of insulin
should be prescribed basing on the following A. Action of bacterial exotoxin
calculation: B. Action of bacterial endotoxin
C. Allergic
A. 0,1-0,2 units/kg of body mass per hour D. Accumulation of suboxidated products
B. 0,05 units/kg of body mass per hour E. Bacteriemia
C. 0,2-0,3 units/kg of body mass per hour
D. 0,3-0,4 units/kg of body mass per hour 67. A 40-year-old woman with a history of
E. 0,4-0,5 units/kg of body mass per hour combined mitral valve disease with predomi-
nant stenosis complains of dyspnea, asthma
63. A 43-year-old female patient complains of attacks at night, heart problems. At present,
rash on the skin of her right leg, pain, weakness, she is unable to do even easy housework. What
body temperature rise up to 38o C . The disease is the optimal tactics of the patient treatment?
is acute. Objectively: there is an edema on the
skin of her right leg in the foot area, a well- A. Mitral commissurotomy
defined bright red spot in the irregular shape B. Implantation of an artificial valve
of flame tips, which is hot to touch. There are C. Antiarrhythmia therapy
isolated vesicles in the focus. What is your D. Treatment of heart failure
provisional diagnosis? E. Antirheumatic therapy
A. Erysipelas 68. A 30-year-old woman with a long history
B. Microbial eczema of chronic pyelonephritis complains of consi-
C. Contact dermatitis derable weakness, sleepiness, decrease in di-
D. Toxicoderma uresis down to 100 ml per day. BP is 200/120
E. Haemorrhagic vasculitis mm Hg. In blood: creatinine - 0,62 millimole/l,
hypoproteinemia, albumines - 32 g/l, potassi-
64. A 47-year-old patient came to see a doctor um - 6,8 millimole/l, hypochromic anemia,
on the 7th day of disease. The disease is increased ESR. What is the first step in the
acute: after the chill body temperature rose patient treatment tactics?
to 40o C and lasted up to 7 hours, then dropped
abruptly, which caused profuse sweat. There
were three such attacks occurring once every
Крок 2 Medicine (англомовний варiант, iноземнi студенти) 2015 рiк 9

A. Haemodialysis 73. A 12-year-old cild has been hit on the


B. Antibacterial therapy stomach. The patient is in moderately grave
C. Enterosorption condition, has a forced position in bed. The
D. Haemosorption skin is pale, heart rate - 122/min. Stress on
E. Blood transfusion the left costal arch causes a slight pain. Wei-
nert and Kulenkampff symptoms are positive.
69. A 43-year-old patient had right-sided deep Macroscopically the urine is unchanged. What
vein thrombosis of iliofemoral segment 3 years is the most likely diagnosis?
ago. Now he is suffering from the sense of
heaviness, edema of the lower right extremi- A. Spleen rupture, abdominal bleeding
ty. Objectively: moderate edema of shin, brown B. Left kidney rupture, retroperitoneal
induration of skin in the lower third of shin, hematoma
varix dilatation of superficial shin veins are C. Rupture of the pancreas
present. What is the most probable diagnosis? D. Liver rupture, abdominal bleeding
E. Rupture of a hollow organ, peritonitis
A. Postthrombophlebitic syndrome, varicose
form 74. Establishments participating in medical
B. Acute thrombosis of right veins examinations include: medical and preventi-
C. Lymphedema of lower right extremity ve treatment facility, hygiene and preventive
D. Parkes-Weber syndrome treatment facility, sociomedical expert commi-
E. Acute thrombophlebitis of superficial veins ttees, Ministry of Defence medical commi-
ttees, Ministry of Domestic Affairs medical
70. A 28-year-old woman has bursting pain committees, forensic medicine agency, etc.
in the lower abdomen during menstruati- Specify what service deals with sociomedical
on; chocolate-like discharges from vagina are assessment of temporary disability:
observed. It is known from the anamnesis that
the patient suffers from chronic adnexitis. Bi- A. Medical and preventive treatment facility
manual examination revealed a tumour-like B. Hygiene and preventive treatment facility
formation of heterogenous consistency 7х7 cm C. Sociomedical expert committees
large to the left from the uterus. The formation D. Ministry of Defence medical committees
is restrictedly movable, painful when moved. E. Ministry of Domestic Affairs medical commi-
What is the most probable diagnosis? ttees
A. Endometrioid cyst of the left ovary 75. An unconscious 35-year-old patient has
B. Follicular cyst of the left ovary been delivered by an ambulance to the intensi-
C. Fibromatous node ve care unit. Objectively: the patient is in
D. Exacerbation of chronic adnexitis semicoma. Moderate mydriasis is present. The
E. Tumour of sigmoid colon reaction of pupils to light is reduced. The pati-
ent is non-responsive to verbal instructions. BP
71. A 6-year-old child complains of frequent li- is150/100 mm Hg, tachycardia is present. Blood
quid stool and vomiting. On the 2nd day of contains methanol. What antidote should be
desease the child presented with inertness, administered?
temperature rise up to 38, 2o C , heart rate -
150 bpm, scaphoid abdomen, palpatory painful A. Ethanol
sigmoid colon, defecation 10 times a day with B. Unithiol
liquid, scarce stool with mucus and streaks of C. Thiamine chloride
green. What is a provisional diagnosis? D. Tavegil (Clemastine)
E. Naloxone
A. Shigellosis
B. Salmonellosis 76. After lifting a load, a 36-year-old male pati-
C. Escherichiosis ent has experienced a severe pain in the lumbar
D. Intestinal amebiasis region, which spread to the right leg and was
E. Yersiniosis getting worse when he moved his foot or
coughed. Objectively: the long back muscles on
72. A 65-year-old patient complains of pain in the right are strained. Achilles jerk is reduced
the lumbar spine, moderate disuria. He has on the right. There is a pronounced tenderness
been suffering from this condition for about of paravertebral points in the lumbar region.
half a year. Prostate volume is 45 cm3 (there The straight leg raise (Lasegue’s sign) is posi-
are hypoechogenic nodes in both lobes, capsule tive on the right. What additional tests should
invasion). Prostate-specific antigen is 60 ng/ml. be performed in the first place?
Prostate biopsy revealed an adenocarcinoma.
Which of the supplementary examination A. Radiography of the spinal column
methods will allow to determine the stage of B. Computed tomography
neoplastic process in this patient? C. Magnetic resonance tomography
D. Electromyography
A. Computer tomography of small pelvis E. Lumbar puncture
B. Lumbar spine X-ray
C. Excretory urography 77. A patient is active, lively, fussy. He resi-
D. Bone scintigraphy sts the examination. His speech is fast and
E. Chest X-ray loud, his statements are spontaneous and
Крок 2 Medicine (англомовний варiант, iноземнi студенти) 2015 рiк 10

inconsequent. Specify the psychopathological diagnosis is:


state of this patient:
A. Uterine rupture
A. Psychomotor agitation B. Initial uterine rupture
B. Catatonic agitation C. Threatened uterine rupture
C. Delirium D. Premature detachment of normally positi-
D. Behavioral disorder oned placenta
E. Paranoid syndrome E. Compression of inferior pudendal vein
78. A 4-year-old boy was vaccinated in vi- 82. A 26-year-old female patient has an 11-
olation of his vaccination schedule. There year history of rheumatism. Four years ago
are complaints of pain in the throat during she suffered 2 rheumatic attacks. Over the last
swallowing, headache, inertness, fever. Objecti- 6 months there have been paroxysms of atri-
vely: the child is pale; anterior cervical lymph al fibrillation every 2-3 months. What option
nodes are enlarged; tonsils are swollen, with of antiarrhythmic therapy or tactics should be
cyanotic hyperemy, and covered with grey- proposed?
white coating that cannot be removed; if it is
forcibly removed, tonsils bleed. The most li- A. Prophylactic administration of cordarone
kely diagnosis is: B. Immediate hospitalization
C. Defibrillation
A. Oropharyngeal diphtheria D. Lidocaine administration
B. Lacunar tonsillitis E. Heparin administration
C. Pseudomembranous (Vincent’s) tonsillitis
D. Infectious mononucleosis 83. A 47-year-old female patient complains of
E. Follicular tonsillitis cough with purulent sputum, pain in the lower
left chest, periodical body temperature rise.
79. A 26-year-old male patient complains of pi- She has been suffering from this condition for
ercing pain during breathing, cough, dyspnea. about 10 years. Objectively: "drumstick"distal
Objectively: to - 37, 3o C , respiration rate is phalanges. What examination would be the
19/min, heart rate is 92/min; BP is 120/80 mm most informative for making a diagnosis?
Hg. Vesicular respiration is observed. In the
inferolateral parts of chest auscultation in both A. Bronchography
inspiration and expiration phase revealed noi- B. Bronchoscopy
se that was getting stronger at phonendoscope C. Survey radiograph of lungs
pressing and could be still heard after cough. D. Pleural puncture
ECG showed no pathological changes. What is E. Bacteriological analysis of sputum
the most likely diagnosis?
84. A patient is 60-year-old, retired, worked
A. Acute pleuritis as deputy director of a research institute.
B. Intercostal neuralgia Behavioural changes appeared 2 years ago
C. Subcutaneous emphysema after the death of her husband: she stopped
D. Spontaneous pneumothorax looking after herself and leaving the house;
E. Pericarditis sicca then she stopped to clean the apartment and
cook. Mental status: temporal disorientation.
80. A child is 3-week-old. Since his birth peri- The patient does not understand many of the
odical vomiting is observed occuring several questions, is confused; does not know how
minutes after feeding. Vomit mass does not to cook soup or fasten a button. Her speech
exceed previous feeding volume. Body mass is is characterized by stumbling and logoclonia.
appropriate to the child’s age. What is the most She does not recognize doctors, fellow pati-
likely cause for the symptoms described? ents. She cries a lot but can not explain the
reason for tears. What is the mechanism of this
A. Pylorospasm pathology?
B. Esophageal chalasia
C. Adrenogenital syndrome A. Atrophy of the cerebral cortex
D. Pylorostenosis B. Atherosclerotic changes in cerebral vessels
E. Esophageal achalasia C. Serotonin deficiency
D. Impaired conversion of dopamine to
81. A 30-year-old parturient woman was deli- noradrenaline
vered to a maternity hospital with full-term E. Disorder of melatonin metabolism
pregnancy. She complains of severe lancinati-
ng pain in the uterus that started 1 hour ago, 85. An 8-year-old girl periodically has sudden
nausea, vomiting, cold sweat. Anamnesis states short-term heart pain, sensation of chest
cesarean section 2 years ago. Uterine contracti- compression, epigastric pain, dizziness, vomiti-
ons stopped. Skin and mucous membranes are ng. Objectively: the patient is pale, respiratory
pale. Heart rate is 100/min, BP is 90/60 mm Hg. rate - 40/min, jugular pulse is present. Heart
Uterus has no clear margins, is sharply painful. rate - 185 bpm, of poor volume. BP - 75/40
No heartbeat can be auscultated in the fetus. mm Hg. ECG taken during an attack shows
Moderate bloody discharge from the uterus ectopic P waves, QRS wave is not deformed.
can be observed. Uterus cervix is 4 cm open. At the end of an attack a compensatory pause
Presenting part is not visible. The most likely is observed. The most likely cause of the attack
Крок 2 Medicine (англомовний варiант, iноземнi студенти) 2015 рiк 11

is: A. Thermograph
B. Alcohol thermometer
A. Paroxysmal atrial tachycardia C. Mercury thermometer
B. Sinus tachycardia D. August’s psychrometer
C. Paroxysmal ventricular tachycardia E. Assmann psychrometer
D. Complete AV-block
E. Atrial fibrillation 91. Against the background of angina a patient
has developed pain in tubular bones. Exami-
86. During examination a patient is unconsci- nation revealed generalized enlargement
ous, his skin is dry and hot, face hyperemia of lymph nodes, hepatolienal syndrome,
is present. The patient has Kussmaul’s respi- sternalgia. In blood: RBCs - 3, 6 · 1012 /l, Hb-
ration, there is also smell of acetone in the 87 g/l, thrombocytes - 45 · 109 /l, WBCs -
air. Symptoms of peritoneum irritation are
positive. Blood sugar is 33 millimole/l. What 13 · 109 /l, blasts - 87%, stab neutrophiles - 1%,
emergency actions should be taken? segmented neutrophiles - 7%, lymphocytes -
5%, ESR - 55 mm/h. What is the most likely
A. Intravenous infusion of short-acting insulin diagnosis?
B. Intravenous infusion of glucose along with
insulin A. Acute leukemia
C. Introduction of long-acting insulin B. Erythremia
D. Intravenous infusion of neohaemodesum C. Chronic lymphocytic leukemia
along with glutamic acid D. Chronic myeloid leukemia
E. Intravenous infusion of sodium chloride E. Multiple myeloma
saline 92. For a long time to eradicate weeds on
87. A patient complains of evaginations in the agricultural lands herbicides retaining in envi-
region of anus that appear during defecation ronment have been used. Point out the most
and need resetting. Examination with anoscope probable way of herbicides reaching human
revealed 1x1 cm large evaginations of mucosa organism from soil:
above the pectineal line. What is the most A. Soil-plants-human
probable diagnosis? B. Soil-microorganisms-human
A. Internal hemorrhoids C. Soil-animals-human
B. Acute paraproctitis D. Soil-protozoa-human
C. External hemorrhoids E. Soil-insects-human
D. Anal fissure 93. A 52-year-old patient complains of pain in
E. - the right part of her chest, dyspnea, cough wi-
88. During the newborn infant’s examinati- th a lot of albuminoid sputum emitting foul
on redness of the skin was detected, which smell of "meat slops". Objectively: the pati-
occurred immediately after birth and became ent’s condition is grave, cyanosis is observed,
the most pronounced on the second day of the breathing rate is 31/min, percussion sound
infant’s life. What provisional diagnosis can be above the right lung is shortened, auscultation
made? revealed various moist rales (crackles). What is
the most probable diagnosis?
A. Simple erythema
B. Toxic erythema A. Lung gangrene
C. Transient erythema B. Lung abscess
D. Erythema nodosum C. Pleura empyema
E. Erythema anulare centrifugum D. Multiple bronchiectasis
E. Chronic pneumonia
89. A baby was born by a young smoker. The
labour was complicated by uterine inertia, diffi- 94. A 15-year-old patient suffers from
cult delivery of the baby’s head and shoulders. headache, nasal haemorrhages, sense of lower
The baby’s Apgar score was 4. Which of the extremity coldness. Objectively: muscles of
following is a risk factor for a spinal cord shoulder girdle are developed, lower extremi-
injury? ties are hypotrophied. Pulsation on the pedal
and femoral arteries is sharply dampened. BP
A. Difficult delivery of the head and shoulders is 150/90 mm Hg, 90/60 in the legs. Systolic
B. Young age of the mother murmur can be auscultated above carotid
C. Pernicious habits arteries. What is the most probable diagnosis?
D. Uterine inertia
E. Chronic hypoxia A. Aorta coarctation
B. Aorta aneurism
90. In order to study impact of microclimate on C. Aortal stenosis
the human organism it is necessary to make D. Aortal insufficiency
systematic observation of air temperature E. Coarctation of pulmonary artery
during the period of 3 days. Choose a devi-
ce that will allow to make the most precise 95. A patient with frostbite of both feet was
temperature records: delivered to an admission ward. What actions
should be taken?
Крок 2 Medicine (англомовний варiант, iноземнi студенти) 2015 рiк 12

A. Disturbed urine outflow from the kidney


A. To apply a bandage, to introduce vasodilating B. Increase in urine specific gravity
medications C. Ureteric twists
B. To administer cardiac medications D. Destruction of renal glomerules
C. To put feet into hot water E. Renal artery spasm
D. To rub feet with snow
E. To apply an alcohol compress 101. A 26-year-old woman has attended
maternity center complaining of her inability
96. A patient in a clinical death condition is bei- to become pregnant despite 3 years of regular
ng resuscitated through mouth-to-mouth artifi- sex life. Examination revealed the followi-
cial pulmonary ventilation and external cardi- ng: increased body weight; male-type pubic
ac massage. A doctor noticed that air does not hair; excessive pilosis of thighs; ovaries are
flow into the patient’s airways and his head and dense and enlarged; basal body temperature
torso are positioned at the same level. Why is is monophasic. The most likely diagnosis is:
artificial respiration not effective in the given
case? A. Ovaries sclerocystosis
B. Inflammation of uterine appendages
A. Tongue retraction C. Adrenogenital syndrome
B. Low breathing volume D. Premenstrual syndrome
C. External cardiac massage E. Gonadal dysgenesis
D. Probe is absent from stomach
E. The patient’s mouth is too small 102. A workshop resident doctor makes a li-
st of workers, who are often ill, for special
97. Hygienic assessment of a sample taken supervision. He takes into account the number
from the batch of grain revealed 2% of grains of etiologically connected cases resulting in
infected with microscopic Fusarium fungi. On temporary disability that occurred in the span
the grounds of laboratory analyses this batch of a year for each worker. How many such cases
of grain should be: should a worker have to be included in this
group?
A. Sold without restrictions
B. Tested for toxicity A. 4 and more
C. Used for forage production B. 1 and more
D. Used for ethanol production C. 2 and more
E. Destroyed D. 3 and more
E. 6 and more
98. Residents of an industrial community si-
tuated near a plant suffer from increased 103. A 28-year-old woman complains of
morbidity rate caused by nervous and endocri- increased intermenstrual periods up to 2
ne system conditions and kidney diseases. months, hirsutism. Gynaecological examinati-
Blood test: decrease of sulfhydric groups on revealed that the ovaries were enlarged,
content in blood. The pathologies developed painless, compact, uterus had no peculariti-
can be caused by environment being polluted es. Pelvic ultrasound revealed that the ovari-
by the following: es were 4-5 cm in diameter and had multiple
enlarged follicles on periphery. X-ray of skull
A. Mercury base showed that sellar region was dilated.
B. Cadmium What is the most probable diagnosis?
C. Boron
D. Chromium A. Stein-Leventhal syndrome (Polycystic ovary
E. Lead syndrome)
B. Algodismenorrhea
99. Various population groups mortality has C. Sheehan’s syndrome
been studied for a long time; territorial distri- D. Premenstrual syndrome
bution of population and resulting changes in E. Morgagni-Stewart syndrome
mortality trends are considered. What statstical
method can be applied? 104. A woman consulted a therapeutist
about fatigability, significant weight loss,
A. Time series analysis weakness, loss of appetite. She has been having
B. Standardized ratio analysis amenorrhea for 8 months. A year ago she born
C. Correlation and regression analysis a full-term child. Haemorrhage during labour
D. Assessment of parameter difference reliabi- made up 2 l. She got blood and blood substi-
lity tute transfusions. What is the most probable
E. Analysis of relative values diagnosis?
100. A 45-year-old patient with urolithiasis had A. Sheehan’s syndrome
an attack of renal colic. What is the mechanism B. Stein-Leventhal syndrome
of the attack development? C. Shereshevsky-Turner’s syndrome
D. Homological blood syndrome
E. Vegetovascular dystonia
105. The correlation between service record
Крок 2 Medicine (англомовний варiант, iноземнi студенти) 2015 рiк 13

and eosinophiles concentration in blood was A. Alimentary constitutive obesity, I stage,


studied in workers at dyeing shops of texti- android type
le factories. What index will be the most B. Itsenko-Cushing hypothalamic obesity, II
informative for the analysis of this data? stage, gynoid type
C. Alimentary constitutive obesity, III stage,
A. Correlation factor gynoid type
B. Student’s criterion D. Alimentary constitutive obesity, II stage,
C. Standardized index android type
D. Fitting criterion E. Itsenko-Cushing hypothalamic obesity, I
E. Sign index stage, android type
106. A 39-year-old woman suffering from schi- 110. A parturient woman is 27 year old, it was
zophrenia constantly strains to hear somethi- her second labour, delivery was at full-term,
ng insisting that "there is a phone connecti- normal course. On the 3rd day of postpartum
on in her brain and she hears her brother’s period body temperature is 36, 8o C , heart rate
voice demanding that she come back home". - 72/min, BP - 120/80 mm Hg. Mammary glands
The patient is anxious, suspicious, constantly are moderately swollen, nipples are clean.
looking around. Specify the psychopathologic Abdomen is soft and painless. Fundus of uterus
syndrome. is 3 fingers below the umbilicus. Lochia are
bloody, moderate. What is the most probable
A. Hallucinatory diagnosis?
B. Anxiety
C. Paranoid A. Physiological course of postpartum period
D. Paraphrenic B. Subinvolution of uterus
E. Depressive C. Postpartum metroendometritis
D. Remnants of placental tissue after labour
107. A 3-month-old girl has rhinitis, dyspnea, E. Lactostasis
dry cough. She has been sick for 2 days.
Objectively: pale skin, acrocyanosis, hypopnoe; 111. A 40-year-old patient suffers from
breathing rate is 80/min; over the whole influenza. On the 5th day of illness there are
pulmonary surface there is vesiculotympanitic pain behind sternum, cough with sputum,
(bandbox) resonance observed with numerous inertness. Temperature is 39, 5o C . Face is
bubbling rales (crackles). The most likely di- pale. Mucosa of conjunctivas and pharynx is
agnosis is: hyperemic. Heart rate is 120/min, breathing
rate is 38/min. In the lower lung segments
A. Acute bronchiolitis shortening of percussion sound and moist rales
B. Pneumonia (crackles) can be detected. What additional
C. Mucoviscidosis investigation should be performed first of all to
D. Foreign body in airways specify the diagnosis?
E. Acute bronchitis
A. Lung X-ray
108. A 30-year-old patient was hospitalised in B. ECG
an intensive care unit with a diagnosis of multi- C. Heart US
ple bee stings. Skin is pale and covered with D. Mantoux test
cold sweat. Pulse can be palpated only at the E. Spirography
carotid arteries and is 110/min; breating rate is
24/min, rhytmical, weakened. What drug must 112. A 45-year-old patient complains of body
be administered immediately? temperature rise up to 40o C , general weakness,
headache, painfulness and spastic muscle
A. Adrenaline hydrochloride intravenously contractions around the wound in the shin. He
B. Prednisolone intravenously received this wound 5 days ago when worki-
C. Adrenaline hydrochloride intramuscularly ng in his garden. He requested no medical
D. Dopamine intravenously care back then. What wound infection can be
E. Tavegyl (Clemastine) intravenously suspected?
109. A 24-year-old patient complains of gaining A. Tetanus
body mass and increased appetite. Objectively: B. Anthrax
built of hypersthenic type, body mass index C. Erysipelas
is 33,2 kg/m2 , waist circumference is 100 cm. D. Gram-positive
Waist to hips circumference ratio is 0,95. What E. Gram-negative
is the provisional diagnosis?
113. A 60-year-old woman started feeling
weakness, vertigo, rapid fatigability during the
last year. Recently there have been dyspnea
and paresthesia observed. Objectively: skin
and mucous membranes are pale and icteric.
Lingual papillae are smoothed out. Liver and
spleen are situated at the edge of costal arch.
Blood test: Hb- 70 g/l, erythrocytes - 1, 7·1012 /l,
blood color index - 1,2, macrocytes. What drug
Крок 2 Medicine (англомовний варiант, iноземнi студенти) 2015 рiк 14

can be prescribed on pathogenetic grounds? A. Acute bartholinitis


B. Furuncle of outer labia
A. Vitamin B12 C. Acute vulvovaginitis
B. Vitamin B6 D. Bartholin’s cyst
C. Ascorbic acid E. Carcinoma of vulva
D. Iron preparations
E. Vitamin B1 118. In a rural health care area there is a case of
child dying during the first month of his life. To
114. A 34-year-old patient complains of profuse analyse this situation, among other measures,
sweating at night, skin itching, weight loss an expert assessment of medical records is
(9 kg within the last 3 months). Examinati- performed. What medical document should be
on revealed malnutrition, skin pallor. Palpati- considered first?
on of neck and inguinal areas revealed dense
elastic lymph nodes of about 1 cm in diameter, A. Child development history
nonmobile, non-adhering to skin. What is the B. Infant development history
most probable diagnosis? C. Vaccination card
D. Outpatient medical record
A. Lymphogranulomatosis E. Child’s medical record
B. Chronic lymphadenitis
C. Lymphosarcoma 119. Examination of placenta revealed a defect.
D. Burkitt’s lymphoma An obstetrician performed manual investi-
E. Cancer metastases gation of uterine cavity, uterine massage.
Prophylaxis of endometritis in the postpartum
115. A 57-year-old woman complains of having period should involve the following actions:
a sensation of esophageal compresion, palpi-
tation, breathing difficulties when eating solid A. Antibacterial therapy
food, occasional vomiting with a full mouth, B. Instrumental revision of uterine cavity
"wet pillow"sign at night for the last 6 months. C. Haemostatic therapy
Objectively: body tempearture - 39o C , height - D. Contracting agents
168 cm, weight - 72 kg, heart rate - 76/min, BP- E. Intrauterine instillation of dioxine
120/80 mm Hg. X-ray revealed considerable di- 120. A 15-year-old boy has developed pain in
lation of esophagus and its constriction in the the area of his left knee joint. Objectively:
cardial part. What pathology is most likely to thigh soft tissues in the painful area are infi-
have caused dysphagia in this patient? ltrated, joint function is reduced. X-ray: there is
A. Achalasia cardiae a focus of destruction in the left distal femoral
B. Primary esophagism metaphysis, with periosteum detachment and
C. Hiatal hernia formation of Codman triangle in the bone
D. Esophageal carcinoma cortical layer at the margin of the defect. Chest
E. Reflux esophagitis X-ray reveals numerous microfocal metastases.
What pathology is the most likely to cause such
116. A 25-year-old man complains of pain presentation?
in the lower third of his left thigh, which
occurs both with and without physical exertion. A. Osteogenic sarcoma
Possibility of trauma is denied by the patient. B. Fibrosarcoma
Objectively: skin colour is normal; pastosity C. Chondrosarcoma
and pain can be detected with deep palpati- D. Ewing’s sarcoma
on; knee joint mobility is reduced. X-ray of E. Juxtacortical sarcoma
distal femoral metaphysis shows an area of 121. A 74-year-old man complains of pain in
destruction and spicule. Blood test: immature the left foot and dark color of the 1st toe. He
cells, no signs of inflammation. The most likely has been sick for 6 days. Objectively: heart
diagnosis is: rate is 84/min. Body temperature is 37, 8o C .
A. Osteogenic sarcoma The left foot resembles "chicken claw"due
B. Osteitis fibrosa cystica to soft tissues decreasing in volume, skin is
C. Chronic osteomyelitis dark red. The 1st toe is black, small amount of
D. Multiple myeloma dark substance is being discharged from under
E. Marble-bone disease the nail. Pulse over the peripheral arteries of
extremity is absent. The most likely diagnosis
117. A 37-year-old woman complains of sharp is:
pains in her external genitalia, edema of the
vulvar lips, pain when walking. Objectively: A. Gangrene
body temperature is 38, 7o C , heart rate is B. Phlegmon
98/min. Inside the right vulvar lip there is a C. Panaritium
dense, painful, tumor-like growth 5,0х4,5 cm in D. Erysipelas
size; skin and mucosa of the external genitalia E. Erysipeloid
are hyperemic, copious foul-smelling discharge 122. Head circumference of a 1-month-old boy
is observed. The most likely diagnosis is: with signs of excitement is 37 cm, prefontanel
is 2x2 cm large. After feeding the child regurgi-
tates small portions of milk; stool is normal
Крок 2 Medicine (англомовний варiант, iноземнi студенти) 2015 рiк 15

in its volume and composition. Muscle tone is


within norm. What is the most likely diagnosis? A. Diabetes mellitus type 2 with visceral
neuropathy
A. Pylorospasm B. Diabetes mellitus type 1 with ketoacidosis
B. Meningitis C. Diabetes mellitus type 1 with
C. Pylorostenosis encephalomyelopathy
D. Microcephaly D. Diabetes mellitus type 2 with nephropathy
E. Craniostenosis E. Diabetes mellitus type 2 with polyneuropathy
123. A 49-year-old patient complains of di- 127. A young woman suffering from seborrhea
fficulties when swallowing food, hoarse voi- oleosa has numerous light-brown and whi-
ce, weight loss. The patient has been suffering te spots on the skin of her torso and
from these symptoms for the last 3 months. The shoulders. The spots have clear margins,
symptoms are gradually progressing. Objecti- branny desquamation, no itching. What provi-
vely: the patient is emaciated, supraclavicular sional diagnosis can be made?
lymph nodes are enlarged. Esophagoscopy
revealed no pathologies of esophagus. What A. Pityriasis versicolor
investigation would be the most advisable in B. Torso dermatophytosis
the given case? C. Seborrheic dermatitis
D. Pityriasis rosea
A. Computed tomogram of chest and mediasti- E. Vitiligo
num
B. Lungs X-ray 128. 10 days after birth an infant developed
C. Multiplanar fluoroscopy of esophagus a sudden fever up to 38, 1o C . Objectively:
D. Radioisotopic examination of chest and the skin of navel, abdomen and chest is
mediastinum erythematous; there are multiple pea-sized
E. US of mediastinum blisters with no infiltration at the base; si-
ngle bright red moist erosions with epidermal
124. A 36-year-old male patient complains of fragments on the periphery. What is your provi-
having headache, obstructed nasal breathing, sional diagnosis?
purulent nasal discharge for 2 weeks. A month
before, he had his right maxillary premolar A. Epidemic pemphigus of newborn
filled. X-ray revealed an intense opacity of B. Syphilitic pemphigus
the right maxillary sinus. Diagnostic puncture C. Streptococcal impetigo
revealed a large amount of thick malodorous D. Vulgar impetigo
crumbly pus. What is the most likely diagnosis? E. Atopic dermatitis

A. Chronic suppurative odontogenic sinusitis 129. In a traffic accident two persons died. An
B. Acute purulent sinusitis appointed forensic medical expert was called
C. Chronic purulent sinusitis on duty to another accident location; therefore,
D. Chronic atrophic sinusitis according to the crininal law in force in Ukrai-
E. Tumor of the maxillary sinus ne, an investigator called in the following speci-
alist for examination of the location and bodi-
125. During a surgical operation necessitated es:
by the patient’s suffering from autoimmune
thyroiditis with concomitant multinodular goi- A. Nearest available medical doctor
ter the right lobe was removed and subtotal B. Medical assistant
resection of the left lobe was performed. C. Surgical nurse
What should be prescribed to the patient for D. Nearest available dispensing chemist
postoperative hypothyroidism prevention? E. Nearest available dental mechanic

A. L-thyroxin 130. A 35-year-old female patient has gained


B. Mercazolil (Thiamazole) 20 kg weight within a year with the normal di-
C. Iodomarin (Potassium iodide) et. She complains of chill, sleepiness, dyspnea.
D. Lithium preparations The patient’s mother and sister are corpulent.
E. Insulin Objectively: height - 160 cm, weight - 92 kg,
BMI - 35,9. Obesity is uniform, there are no
126. A 54-year-old patient has been suffering striae. The face is amimic. The skin is dry. The
from diabetes mellitus for 5 years, with diet tongue is thickened. Heart sounds are muffled.
being his only treatment. Within the last half Heart rate - 56/min, BP - 140/100 mm Hg. The
a year he lost 7 kg of body weight, complai- patient has constipations, amenorrhea for 5
ns of thirst, vertigo when raising from bed, months. TSH - 28 mkME/l (normal rate - 0,32-
decrease of erectile function, frequent stool, 5). Craniogram shows no pathology. What is
especially at night. Objectively: malnutrition, the etiology of obesity?
dry skin. BP in lying position is 160/90 mm
Hg; BP in standing position is 170/85 mm Hg. A. Hypothyroid
No edemas. Fasting plasma glucose level is B. Hypo-ovarian
12 mmol/l. Glycated hemoglobin accounts for C. Hypothalamic-pituitary
11%. Albumin excreted with urine is 20 mg per D. Alimentary and constitutive
day. The most likely diagnosis is: E. Hypercorticoid
Крок 2 Medicine (англомовний варiант, iноземнi студенти) 2015 рiк 16

131. A 45-year-old patient has been sufferi- A. Ileocecal invagination


ng from essential hypertension for 10 years. B. Gastrointestinal haemorrhage
After hot bath he suddenly developed sharp C. Wilm’s tumour
headache and recurrent vomiting. Objecti- D. Helminthic invasion
vely: pronounced meningeal symptoms; BP E. Pylorostenosis
is 180/110 mm Hg. He was hospitalised in a
neurology unit. What additional tests should 135. A 68-year-old patient complains of
be primarily prescribed? pain, slight swelling and creaking in the di-
stal joints of her fingers and knee joints.
A. Lumbar puncture, cerebrospinal fluid test She has been suffering from this condition
B. Ventriculopuncture for 4 years. Objectively: thickening of knee
C. Echoencephalography and interphalangeal joints, restricted mobi-
D. Rheoencephalography lity of these joints. Blood test: leukocytes -
E. Electroencephalography 5, 4 · 109 /l, ESR - 12 mm/h, fibrinogen - 3,5
g/l; Rheumatoid factor - 1:32; anti-streptolysin
132. A 77-years-old patient complains of inabi- O - 160 units; seromucoid - 0,20 units.; CRP+.
lity to urinate and bursting pain above his What mechanism of development is likely for
pubis. Acute onset of his condition occurred this condition?
12 hours ago. Objectively: overfilled urinary
bladder can be palpated above the pubis. On A. Glycosaminoglycanes synthesis disruption
rectal examination: prostate is enlarged, dense- B. Rheumatoid factor development
elastic, with clear margins and no nodules. C. Native DNA antibodies hyperproduction
Interlobar sulcus is pronounced. US examinati- D. Increased uric acid sunthesis
on: prostate volume is 120 cm3 , it protrudes E. Immune reaction to streptococcosis
into urinary bladder cavity, parenchyma is
homogeneous. Prostate specific antigen is 5 136. A 30-year-old male patient consulted a
ng/ml. What disease is the most likely to cause family doctor 2 months after he had been
acute urinary retention? operated for an open fracture of brachial bone.
Objectively: the patient’s condition is sati-
A. Prostatic hyperplasia sfactory; in the area of the operative wound
B. Prostate cancer there is a fistula with some purulent discharge,
C. Prostate sclerosis redness, fluctuation. X-ray image shows brachi-
D. Prostate tuberculosis al bone destruction with sequestra. What
E. Acute prostatitis complication arose in the postoperative peri-
od?
133. A patient with uterine fibromyoma si-
zed up to 8-9 weeks of pregnancy consulted A. Posttraumatic osteomyelitis
a gynaecologist about acute pain in the lower B. Hematogenic osteomyelitis
abdomen. Examination revealed pronounced C. Wound abscess
positive symptoms of peritoneal irritation, hi- D. Posttraumatic phlegmon
gh leukocytosis. Vaginal examination revealed E. Suture sinus
that the uterus was enlarged up to 9 weeks of
pregnancy due to the fibromatous nodes, one 137. A 40-year-old female patient complains of
of which was mobile and extremely painful. having a bulge on the anterior surface of neck
Appendages were not palpable. Discharges for 5 years. Objectively: Heart rate - 72 bpm,
were mucous, coming in moderate amounts. arterial pressure - 110/70 mm Hg, in the right
What is the treatment tactics? lobe of thyroid gland palpation reveals a mobi-
le 4x2 cm node, the left lobe is not palpable, the
A. Urgent surgery (laparotomy) basal metabolic rate is 6%. What is the most li-
B. Surveillance and spasmolytic therapy kely diagnosis?
C. Fractional diagnostic curettage of the uterine
cavity A. Nodular euthyroid goiter
D. Surgical laparoscopy B. Nodular hyperthyroid goiter
E. Surveillance and antibacterial therapy C. Riedel’s thyroiditis
D. Mixed euthyroid goiter
134. A 4-month-old child was admitted to E. The median cervical cyst
a surgical department 8 hours after the first
attack of anxiety. The attacks happen every 138. Estimation of community health level
10 minutes and last for 2-3 minutes, vomi- involved analysis of a report on diseases regi-
ting occurred once. Objectively: the child’s stered among the population of catchment area
condition is grave. Abdomen is soft, palpati- (reporting form 12). What index is calculated
on reveals a tumour-like formation in the right basing on this report?
iliac area. After rectal examination the doctor’s A. Common morbidity rate
finger was stained with blood. What is the most B. Index of pathological affection
probable diagnosis? C. Index of morbidity with temporary disability
D. Index of hospitalized morbidity
E. Index of basic non-epidemic morbidity
139. A tractor driver with the record of servi-
Крок 2 Medicine (англомовний варiант, iноземнi студенти) 2015 рiк 17

ce of 24 years has undergone palestesiometry vely: tuberous slightly movable lump can be
test (Vibration Sensitivity Measurement). palpated in the right subcostal area; veins
Test revealed increased vibration sensitivity of spermatic cord and scrotum are dilated.
threshold at the frequencies of 63-125-259 Hz Blood test: Hb - 86 g/l, ESR - 44 mm/h. Uri-
to 25 dB. Dynamometry is 20 kg on the right ne test: specific gravity - 1020, proteine - 0,99
and 16 kg on the left. Cold stimulus test is posi- g/l, erythrocytes - all field of vision, leukocytes
tive, time of hand temeperature restoration is - 4-6 in the field of vision. The provisional di-
52 minutes. Blanching at pressure symptom is agnosis is:
positive and equals 21. Hypesthesia of upper
and lower limbs is observed and can be classifi- A. Kidney tumor
ed as "gloves"and "socks"polyneuritic pattern. B. Urolithiasis
Make the provisional diagnosis. C. Acute pyelonephritis
D. Acute glomerulonephritis
A. Vibration disease, I stage, caused by combi- E. Nephroptosis
ned vibration
B. Vibration disease, I stage, caused by local 144. A 58-year-old patient complains of
vibration headache in the occipital region, nausea, choki-
C. Vibration disease, II stage, caused by combi- ng, opplotentes. The presentations appeared
ned vibration after a physical exertion. Objectively: the pati-
D. Vibration disease, II stage, caused by local ent is excited. Face is hyperemic. Skin is pale.
vibration Heart sounds are regular, the 2nd aortic sound
E. Vibration disease, I stage, caused by general is accentuated. BP - 240/120 mm Hg, heart
vibration rate - 92/min. Auscultation reveals some fine
moist rales (crackles) in the lower parts of the
140. A patient’s X-ray image (anteroposterior lungs. Liver is not enlarged. ECG shows signs
projection) shows deformation of lung pattern, of hypertrophy and left ventricular overload.
pneumofibrosis, reticular (honeycomb) lung What is the most likely diagnosis?
pattern of lower pulmonary segments, cylindric A. Complicated hypertensic crisis
and fusiform lumps. The most likely diagnosis B. Acute myocardial infarction, pulmonary
is: edema
A. Multiple bronchiectasis C. Bronchial asthma exacerbation
B. Right lung abscess D. Uncomplicated hypertensic crisis
C. Non-hospital-acquired pneumonia E. Community-acquired pneumonia
D. Lungs maldevelopment 145. A 37-year-old patient complains of pain
E. Diffuse pulmonary fibrosis in the lumbar spine, which increases during
141. A 41-year-old patient with Addi- walking, limited mobility, edema of the right si-
son’s disease had influenza. After that he de of abdomen. Focal tuberculosis is recorded
developed adynamia, depression, nausea, in the anamnesis. X-ray shows destruction
vomiting, diarrhea and hypoglycemia. BP of adjacent surfaces of the bodies of the 1-
is 75/50 mm Hg. Blood test: decreased 2 lumbar vertebrae, vertebral bodies height
content of corticosterone, hydrocortisone, is decreased, intervertebral fissure cannot be
13-oxycorticosteroids, 17-oxycorticosteroids. detected. Abdominal US reveals a growth
What condition has developed in the patient? 15x20 cm in size in the retroperitoneal space
and echo signs of liquid. What diagnosis can be
A. Acute adrenal gland insufficiency suspected?
B. Acute gastritis
C. Acute enterocolitis A. Lumbar tuberculous spondylitis
D. Collapse B. Fracture of the 1-2 lumbar vertebrae body
E. Diabetes mellitus C. Metastatic damage of vertebral column
D. Lumbar spondylolisthesis
142. After a contact with chemicals a plant E. Osteochondrosis
worker has suddenly developed stridor, voi-
ce hoarseness, barking cough, progressi- 146. A 45-year-old patient (14-year-long work
ng dyspnea. Objective examination reveals record as a house painter) upon the contact
acrocyanosis. What is your provisional di- with synthetic paint develops skin reddeni-
agnosis? ng, edema, severe itching and oozing lesions
on her face. Symptoms disappear after the
A. Laryngeal edema contact with this chemical substance stops
B. Laryngeal carcinoma but even the smell of paint alone is enough
C. PATE to make them reappear each time. Each
D. Pulmonary atelectasis recurrence is characterised by increased severi-
E. Pneumothorax ty of symptoms. What provisional diagnosis can
be made?
143. A 58-year-old patient complains of general
weakness, weight loss up to 10 kg within the
last 1,5 months, progressive pain in the small
of the back, raise of blood pressure to 220/160
mm Hg, subfebrile body temperature. Objecti-
Крок 2 Medicine (англомовний варiант, iноземнi студенти) 2015 рiк 18

A. Professional eczema A. Pyloric stenosis


B. Simple contact dermatitis B. Penetration of gastric ulcer
C. Allergic contact dermatitis C. Functional pyloric spasm
D. Urticaria D. Foreign body in the stomach (bezoar)
E. Toxicodermia E. Malignization of gastric ulcer
147. In cold season a patient was delivered to 151. A 5-year-old child has body temperature
a hospital. He was rescued from drowning in a risen up to febrile numbers, suffers from
body of water. There was no contact of his ai- inertness, weakness. Examination revealed
rways with water. The patient is anxious, pale, hemorrhage on the skin of limbs and torso.
complains of pain and numbness of limbs, has Enlargement of cervical and axillary lymph
cold shivering. Breathing rate is 22/min, BP nodes can be detected. The liver is 4 cm below
is 120/90 mm Hg, heart rate is 110/min, rectal the costal arch; the spleen is 6 cm below
temperature is 34, 5o C . What kind of warming the costal arch. Blood test: erythrocytes -
should be prescribed to the patient? 2, 3 · 1012 /l, Hb- 60 g/l, platelets - 40 · 109 /l,
A. Passive warming leukocytes - 32, 8 · 109 /l, eosinophiles - 1%,
band neutrophiles - 1%, segmented neutrophi-
B. Infusion of 37O c solutions les - 12%, lymphocytes - 46%, monocytes - 1%,
C. Hot compresses blasts - 40%, Duke’s bleeding time test result is
D. Heat bath 9 min. What examination is necessary to make
E. Hemodialysis with blood warming the diagnosis?
148. Survey radiograph of a 52-year-old worker A. Myelogram
of an agglomeration plant (28 years of experi- B. Lymph nodes biopsy
ence, the concentration of metal dust is 22-37 C. US of abdominal cavity
mg/m3 ) shows mildly pronounced interstitial fi- D. Detection of hepatitis markers
brosis with diffused contrast well-defined small E. Investigation of platelets dynamic functions
nodular shadows. The patient has no complai-
nts. Pulmonary function is not compromised. 152. A child from primipregnancy was born in
What is the provisional diagnosis? a term labor and has body weight of 4000 g
and body length of 57 cm. When born, he was
A. Siderosis nonresponsive to examination. Diagniosis is di-
B. Silicosis ffuse. Heart rate is 80/min. What resuscitation
C. Anthraco-silicatosis measures should be prescribed?
D. Silicatosis
E. Anthracosis A. Begin ALV with mask
B. Introduce 100% oxygen
149. A week before, a 65-year-old male patient C. Intubate and begin ALV
suffered an acute myocardial infarction. His D. Tactile stimulation
general condition has deteriorated: he complai- E. Administer naloxone
ns of dyspnea at rest, pronounced weakness.
Objectively: edema of the lower extremities, 153. A 20-year-old patient was delivered to a
ascites is present. Heart borders are extended, surgical unit complaining of an incised wound
paradoxical pulse is 2 cm displaced from the on his right forearm that has been bleeding
apex beat to the left. What is the most likely for 1,5 days. Suffers from general weakness,
diagnosis? vertigo, cold sweat, opplotentes. Skin and visi-
ble mucous membranes are pale. Heart rate is
A. Acute cardiac aneurysm 110/min, BP is 100/70 mm Hg. Blood test: Hb
B. Recurrent myocardial infarction is 100 g/l, erythrocytes 2, 5 · 1012 /l. What is the
C. Acute pericarditis cause for the paient’s general condition?
D. Cardiosclerotic aneurysm
E. Pulmonary embolism A. Posthemorrhagic anemia
B. Aplastic anemia
150. A 60-year-old patient complains of nearly C. Wound infection
permanent sensation of heaviness and fullness D. Concomitant disease
in the epigastrium, that increases after eating, E. Acute trombophlebitis
foul-smelling eructation, occasional vomiting
with food consumed 1-2 days ago, weight loss. 154. A 20-year-old parturient woman has the I
12 years ago he was first found to have an ulcer labor stage. The pregnancy is full-term. Labors
of pyloric channel. The patient has taken rani- occur every 3 minutes and last for 55 seconds.
tidine for periodic hunger pain. The patient’s Fetus presentation is polar, the head is pressed
condition has been deteriorating over the last to the small pelvis entrance. Heart rate of the
3 months. Objectively: splashing sound in the fetus is 150/min, distinct and rhythmic. Vagi-
epigastrium is present. What kind of compli- na examination: uterus cervix is smoothed out;
cation is it? mouth of the womb is 2 cm open; fetal bladder
is intact; the head is presented over the I plane
of small pelvis; moderate mucous-bloody di-
scharge is observed. What phase of the I labor
stage is it?
Крок 2 Medicine (англомовний варiант, iноземнi студенти) 2015 рiк 19

A. Latent A. Echocardiography
B. Active B. Electrocardiography
C. Slowing-down C. X-ray
D. Physiological preliminary period D. Phonocardiography
E. Primary uterine inertia E. Rheography
155. A 23-year-old woman after stress has 159. A 64-year-old patient has been hospitali-
developed thirst, polydipsia, polyuria, weight sed with complaints of progressive jaundice
loss, increasing fatigue. Later she developed that developed over 3 weeks ago without pain
nausea and somnolence, lost consciousness syndrome, general weakness, loss of appetite.
and was hospitalised. Glycemia is 27 mmol/l, Objectively: temperature is 36, 8o C , heart rate
acetone in urine is sharply positive. Treatment is 78/min, abdomen is soft and painless, peri-
for ketoacidotic coma was initiated. When toneum irritation symptoms are not detected,
would it be advisable to start preventive palpation reveals sharply enlarged tense
treatment of hypoglycemia by introduction of gallbladder. What disease can be characteri-
5% glucose solution? sed with these symptoms?
A. After glycemia rate drops to 13-14 mmol/l A. Cancer of pancreas head
B. 2 hours after beginning of insulinotherapy B. Duodenal ulcer
C. When patient becomes conscious C. Acute cholecystitis
D. After glycemia rate becomes normal D. Chronic cholecystitis
E. If glycemia decreases with the rate over 5 E. Cholecystitis caused by lambliasis
mmol/l per hour
160. A 22-year-old woman on a reduced diet,
156. A 53-year-old woman complains of weight vegetarian, attended a hospital with complai-
loss up to 10 kg within the last 2 years, liquid nts of distorted smell and taste perception and
foul-smelling stool two times a day that poorly lesions in the angles of her mouth. Objecti-
washes off the toilet, periodic bouts of nausea, vely: sclera is distinctly blue. Diagnosis: iron-
girdle pain in the upper abdomen. Objecti- deficiency anemia. What clinical syndrome is
vely: pain in Gubergrits zone (on the right from expressed primarily?
navel) and Mayo-Robson’s point. Biochemical
blood analysis: glucose - 3,2 mmol/l, bilirubin - A. Sideropenic
16,5 micromole/l, crude protein - 56,4 g/l. Urine B. Anemic
diastase - 426 g/h/l. D-xylose test (oral admi- C. Hemorrhagic
D. Hemolytic
nistration of 25 g of d-xylose) after 5 hours E. Myelodysplasic
reveals 3 g of xylose in urine. The most likely
diagnosis is: 161. A 15-year-old patient complains of
excessive body weight, headache, irritability,
A. Pancreatitis. Malabsorption syndrome rapid fatigability. Significant increase of body
B. Pseudomembranous colitis weight occurred at the age of 14. Objectively:
C. Nonspecific ulcerative colitis weight is 90 kg; height is 160 sm, proportional
D. Irritable bowel syndrome body built. Fatty tissue is distributed evenly.
E. Chronic gastritis
There are thin pink striae (stretch marks) on
157. A 23-year-old patient after intake of brake the thighs, abdomen and mammary glands. BP
fluid has developed anuria that has been lasting - 145/90 mm Hg. Provisional diagnosis is:
for 5 days already. Creatinine level increased up
to 0,769 mmol/l. What treatment tactics should A. Pubertate dyspituitarism
be chosen in the given case? B. Alimentary constitutive obesity
C. Somatoform autonomic dysfunction
A. Hemodialysis D. Itsenko-Cushing’s disease
B. Detoxification therapy E. Cushing’s syndrome
C. Antidotal therapy
D. Diuretics 162. A 6-year-old girl attended a general practi-
E. Plasmapheresis tioner with her mother. The child complains
of burning pain and itching in her external
158. A 15-year-old teenager has undergone genitalia. The girl was taking antibiotics the
medical examination in military recruitment day before due to her suffering from acute
center. The following was revealed: interval bronchitis. On examination: external genitalia
systolic murmur at the cardiac apex, accent are swollen, hyperemic, there is white deposit
of the II heart sound over the pulmonary accumulated in the folds. The most likely di-
artery, tachycardia. What additional examinati- agnosis is:
on method will be the most informative for
determining diagnosis? A. Candidal vulvovaginitis
B. Trichomoniasis
C. Nonspecific vulvitis
D. Helminthic invasion
E. Herpes vulvitis
163. A 44-year-old man complains of dyspnea
Крок 2 Medicine (англомовний варiант, iноземнi студенти) 2015 рiк 20

with sensation of lack of air on inhale, palpi- are detected, the skin covering them is bluish-
tations occurring during slight physical exerti- red in colour. Some nodules have fistulous
on, and shin edemas that appear in eveni- openings producing thick purulent mass. Body
ng and resolve in morning. His condition has temperature is 38, 5O c, general malaise. The
been lasting for 5 months already, deteriorati- most likely diagnosis is:
on is gradual. What method of instrumental
diagnostics allows to verify decrease of systolic A. Hydradenitis
function in this patient? B. Carbuncle
C. Cutaneous tuberculosis
A. Echocardiography D. Necrotizing ulcerative trichophytosis
B. Electrocardiography E. Pyoderma chancriformis
C. Computed tomogram
D. Phonocardiography 168. A patient complains of weight gain, chill,
E. Holter blood pressure monitoring edemas, xeroderma, somnolence, difficulties
with focusing. Objectively: height is 165 cm;
164. When playing in a kindergarten a 3- weight is 90 kg; body proportions are of female
year-old child sudenly developed dyspnea, type, to - 35, 8o C , heart rate - 58/min, BP -
paroxysmal compulsive dry cough. The face 105/60 mm Hg. Heart sounds are weakened,
is cyanotic, the eyes are tearful. Vomiting bradycardia is observed. Other internal organs
occurred several times. Breathing is weakened have no changes. Thyroid gland cannot be
over the whole right side of the chest. The palpated. Milk secretion from mammary
provisional diagnosis is: glands is observed. Hormone investigation
revealed increased levels of thyroid-stimulating
A. Foreign body
B. Obstructive bronchitis hormone (TSH) and prolactin, and decreased
C. Bronchial asthma level of thyroxine (T4). Which one is the cause
D. Hysteria fit for obesity?
E. Stenosing laryngotracheitis A. Primary hypothyroidism
165. A patient with otopyosis is in sharply B. Secondary hypothyroidism
deteriorating condition: he developed C. Prolactinoma
headache, vomiting, febrile temperature, D. Hypopituitarism
general hyperesthesia. Meningeal symptoms E. Adiposogenital dystrophy
and stagnant optic disks are observed. There 169. A 54-year-old patient complains of
is no focal symptoms. Liquor is turbid, blood weakness, jaundice, itching skin. Disease onset
pressure is high, albuminocytological dissoci- was 1,5 months ago: fever up to 39o C appeared
ation occurs with neutrophils. What disease at first, with progressive jaundice developed 2
can be suspected? weeks later. On hospitalisation jaundice was
A. Secondary purulent meningitis severely progressed. Liver cannot be palpated.
B. Meningoencephalitis Gallbladder is enlarged and painless. Blood
C. Serous meningitis bilirubin is 190 micromole/l (accounting for
D. Primary purulent meningitis direct bilirubin). Stool is acholic. What is the
E. Subarachnoid hemorrhage most likely jaundice genesis in this patient?

166. A 25-year-old patient is not married and A. Mechanical jaundice


has sexual relations with several partners. Duri- B. Hepatocellular jaundice
ng the last 3 months he noticed small amount of C. Hemolytic jaundice
mucoserous secretions produced from urethra. D. Caroli syndrome
Subjectively: periodical itching or burning pain E. Gilbert’s syndrome
in urethra. Two months ago pain in knee join
developed. Possibility of trauma or exposure 170. A 22-year-old patient complains
to cold is denied by the patient. During the of 8-months-long delay of menstruation.
last week eye discomfort is noted - lacrimation Anamnesis: menarche since the age of 12,5.
and itching. What provisional diagnosis can be Since the age of 18 menstruations are irregular.
suggested? No pregnancies. Mammary glands have normal
development; when the nipples are pressed,
A. Reactive arthritis milk drops are discharged. On gynecological
B. Rheumatoid arthritis examination: moderate uterus hypoplasia. On
C. Seasonal pollinosis hormonal examination: prolactin level exceeds
D. Bacterial nonspecific urethral conjunctivitis the norm two times. On computed tomogram
E. URTI with conjunctiva and joints affected of the sellar region: a space-occupying lesion
4 mm in diameter is detected. The most likely
167. A 19-year-old patient complains of severe diagnosis is:
pain in axillary crease. Condition onset was a
week ago after her swimming in cold river and
epilation. The next day painful "boil"appeared
that was becoming larger every day and
became a plum-sized tumor. Upon examinati-
on nodular conical growths joined together
Крок 2 Medicine (англомовний варiант, iноземнi студенти) 2015 рiк 21

A. Pituitary tumor 175. An emergency doctor has diagnosed a 32-


B. Lactation amenorrhea year-old woman with generalized convulsive
C. Stein–Leventhal syndrome (Polycystic ovary status epilepticus. The deterioration in the pati-
syndrome) ent’s condition is caused by a sudden gap in the
D. Sheehan’s syndrome epilepsy treatment. Specify the doctor’s further
E. Cushing’s disease tactics:
171. A 2-year-old child in a satisfactory conditi- A. Hospitalization in the intensive care unit
on periodically presents with moderate protei- B. Hospitalization in the department of
nuria, microhematuria. US results: the left ki- neurology
dney is undetectable, the right one is enlarged, C. Hospitalization in the department of
there are signs of double pyelocaliceal system. neurosurgery
What investigation is required to specify the D. Outpatient monitoring by a neuropathologist
diagnosis? E. Outpatient monitoring by a neurosurgeon
A. Excretory urography 176. A 19-year-old patient complains of
B. Micturating cystography dyspnea during physical exertion. He often has
C. Retrograde urography bronchitis and pneumonia. Cardiac murmur
D. Doppler study of renal vessels has been observed since his childhood. On
E. Radioisotope renal scan auscultation: there is splitting of II heart sound
over pulmonary arteria, systolic murmur in the
172. For the last 3 years a 12-year-old boy has third intercostal space near the left sternum
been suffering from stomachache, abdominal margin. ECG test shows right bundle-branch
distension, nausea, periodical liquid fatty stool, block. What is the provisional diagnosis?
grey in color, with rotten smell. On palpation:
pain in the epigastrium, Desjardins’ pancreatic A. Interatrial septum defect
point and Chauffard’s triangle; positive Mayo- B. Open arterial duct
Robson’s sign. Insufficiency of pancreas exocri- C. Coarctation of aorta
ne function is suspected. What method is the D. Aortic stenosis
most informative for pancreas exocrine functi- E. Mitral valve insufficiency
on assessment?
177. A 30-year-old patient was in a car accident.
A. Detection of elastase-1 in feces He is unconscious, pale, has thready pulse. In
B. Determining tripsin content in blood serum the middle third of the right thigh there is
C. Pancreas echography an extensive laceration with ongoing profuse
D. Determining amylase content in blood and external arterial bleeding. What urgent actions
urine must be taken to save the life of the patient?
E. Scatological test
A. Tourniquet above the wound of the right
173. A 34-year-old patient was delivered to thigh
a first-aid center with open fracture of the B. Tourniquet below the wound of the right
lower leg bones. Upon examination bleeding thigh
is stated: blood flows in pulsating bursts. What C. Artificial lung ventilation
actions should be taken at this stage of medical D. Precordial thump
aid? E. Plaster bar
A. Apply tourniquet to the thigh proximally to 178. A 75-year-old male patient complains of
the place of hemorrhage and deliver the patient slight pain in the right iliac region. The abdomi-
to operating room nal pain arose 6 days ago and was accompani-
B. Provide immobilization and deliver the ed by nausea. Surgical examination revealed
patient to a hospital moist tongue, heart rate - 76 bpm. BP - 130/80
C. Apply tourniquet to the thigh distally to the mm Hg. Abdomen is soft, slightly painful in
place of hemorrhage and deliver the patient to the right iliac region on deep palpation, the
operating room symptoms of the peritoneum irritation are
D. Apply pressure dressing to stop hemorrhage doubtful. In blood: RBCs - 4, 0 · 1012 /l, Hb-
and deliver the patient to operating room 135 g/l, WBCs - 9, 5 · 109 /l, stab neutrophiles
E. Deliver the patient to operating room - 5%, segmentonuclear - 52%, lymphocytes -
174. A 26-year-old patient with affective bi- 38%, monocytes - 5%, ESR - 20 mm/h. Specify
polar disorder has developed a condition mani- the doctor’s further tactics:
fested by mood improvement, behavioural A. Emergency operation for acute appendicitis
and sexual hyperactivity, verbosity, active body B. Hospitalization, dynamic surveillance
language, reduced need for sleep. Which of the C. Send the patient home
following drugs are most effective in this case? D. Refer the patient to a district therapist
A. Neuroleptics with a sedative effect E. Administration of additional examination:
B. Antidepressants with an activating effect abdominal ultrasound, x-ray contrast study of
C. Neuroleptics with an activating effect the gastrointestinal tract
D. Tranquilizers 179. On the 10th day postpartum a puerperant
E. Antidepressants with a sedative effect woman complains of pain and heaviness in
Крок 2 Medicine (англомовний варiант, iноземнi студенти) 2015 рiк 22

the left mammary gland. Body temperature is 183. A 70-year-old patient consulted a doctor
38, 8o C , Ps- 94 bpm. The left mammary gland is about arrhythmic cardiac activity, dyspnea.
edematic, the supero-external quadrant of skin Objectively: BP - 150/90 mm Hg, extrasystole
is hyperemic. Fluctuation symptom is absent. arrhythmia (10-12 beats per minute), left
The nipples discharge drops of milk when ventricular systolic dysfunction (ejection fracti-
pressed. What is a doctor’s further tactics? on at the rate of 42%). Which of anti-
arrhythmic drugs should be administered as
A. Antibiotic therapy, immobilization and initial therapy in this case?
expression of breast milk
B. Compress to both mammary glands A. Amiodarone
C. Inhibition of lactation B. Flecainide
D. Physiotherapy C. Encainide
E. Opening of the abscess and drainage of the D. Moracizine
mammary gland E. Digoxin
180. An 8-year-old child was hospitalized 184. A 45-year-old male patient complains of
for fever up to 39, 8o C , inertness, moderate acute pain in his right side irradiating to the
headache, vomiting. Examination revealed right thigh and crotch. The patient claims also
meningeal symptoms. Lumbar puncture was to have frequent urination with urine which
performed. The obtained fluid had raised resembles a "meat slops"in color. The patient
opening pressure, it was transparent, with has no previous history of this condition. There
the cell count of 450 cells per 1 mcL (mai- is costovertebral angle tenderness on the right
nly lymphocytes - 90%), glucose level of 2,6 (positive Pasternatsky’s symptom). What is the
mmol/l. What causative agent might have most likely diagnosis?
caused the disease in the child?
A. Urolithiasis
A. Enterovirus B. Acute appendicitis
B. Meningococcus C. Acute pyelonephritis
C. Koch’s bacillus D. Acute cholecystitis. Renal colic
D. Staphylococcus E. Acute pancreatitis
E. Pneumococcus
185. A 38-year-old man works within the
181. A 25-year-old patient during self- range of ionizing radiation. At a routine medi-
examination detected a tumor in the upper cal examination he presents no problems. In
external quadrant of the right mammary gland. blood: RBCs - 4, 5 · 1012 /l, Hb- 80 g/l, WBCs -
On palpation: painless, dense, mobile growth 2
cm in diameter is detected in the mammary 2, 8 · 109 /l, thrombocytes - 30 · 109 /l. Decide if
gland; no changes in the peripheral lymph this person can work with sources of ionizing
nodes are observed. On mammary glands US: radiation:
in the upper external quadrant of the right A. Working with radioactive substances and
mammary gland there is a space-occupying other sources of ionizing radiation is contraindi-
lesion of increased echogenicity 21х18 mm in cated
size. The most likely diagnosis is: B. The patient is allowed to work with radioacti-
A. Fibrous adenoma ve substances
B. Lacteal cyst C. The patient can only work with radioactive
C. Diffuse mastopathy substances of low activity
D. Breast cancer D. The patient can be allowed to work after an
E. Mastitis extended medical examination
E. The patient is allowed to work with radioacti-
182. A 20-year-old woman complains of feeling ve substances for the limited period of time
of air shortage, lingering dull pain in the heart
area, irritability. Objectively: general condition 186. A maternity patient breastfeeding for 1,5
is satisfactory, heart rate lability, BP is 130/60 weeks has attended a doctor. She considers
mm Hg. ECG reveals disruption of repolari- the onset of her disease to be when proporti-
zation proceses. The patient is diagnosed with onal breast engorgement occurred. Mammary
somatoform autonomic dysfunction of cardi- glands are painful. Body temperature is
al type. Specify the conditions of the patient 36, 6o C . Expression of breast milk is hindered.
treatment: The most likely diagnosis is:

A. Out-patient treatment A. Lactostasis


B. In-patient treatment in therapeutics B. Infiltrative mastitis
department C. Suppurative mastitis
C. In-patient treatment in cardiology D. Chronic cystic mastitis
department E. Gangrenous mastitis
D. In-patient treatment in cardiac surgery 187. A 12-year-old girl complains of general
department weakness, rise of body temperature up to
E. In-patient treatment in psychiatric 38, 2o C , pain and swelling of knee joints, feeli-
department ng of cardiac rhythm disruption. The chi-
Крок 2 Medicine (англомовний варiант, iноземнi студенти) 2015 рiк 23

ld had tonsillitis 3 weeks ago. The knee joi- ultrastructural change of ciliated epithelium.
nts are swollen, local raise of temperature is What is the basis of the given syndrome?
observed, mobility is reduced. Heart sounds
are weakened, extrasystole is present; at the A. Primary ciliary diskinesia
cardiac apex systolic noise can be auscultated, B. Proteoglycans insufficiency
which is not conducted to the left axillary regi- C. Surfactant deficit
on. ESR is 38 mm/h. CRP 2+. Antistreptolysin- D. Muscle cells atony
O titers are 400. The most likely disease is: E. Alpha-1-antitrypsin deficiency

A. Acute rheumatic fever 192. A newborn infant (the first labor, lasted
B. Somatoform autonomic dysfunction for 26 hours) is 1-day-old, postmature; body
C. Non-rheumatic carditis weight is 3850 g; body length is 52 cm. Deli-
D. Juvenile rheumatoid arthritis very was performed by applying obstetrical
E. Reactive arthritis forceps in sincipital presentation, Apgar score
is 1/3. The face is bluish-pale. The head is
188. A 60-year-old man has a diet consisting of thrown back; severe birth trauma is present; the
unvaried food staples: mostly cereals, potato, infant is excitable, shrill "cerebral scream"is
pasta; few vegetables and little fats (especially present; the eyes are half-open; facial expressi-
animal fats). During medical examination he on is attentive; hyperesthesia, hypersthenia and
complains of deterioration of his twilight visi- readiness for convulsions are present. Liquor
on. This condition can be caused by lack of: has high content of erythrocytes, lymphocytic
cytosis occurs. The most likely diagnosis is:
A. Retinol
B. Amino acids A. Subarachnoid hemorrhage
C. Fats B. Epidural hemorrhage
D. Calcium C. Subdural hemorrhage
E. Carbohydrates D. Intraventricular hemorrhage
E. Intracerebral hemorrhage
189. A 45-year-old patient was hospitalised
in surgical in-patient unit with intermuscular 193. A 74-year-old patient visited a urologist
phlegmon of the right thigh on the 6th day from with complaints of pain above the pubis and
the onset of disease. Abscess formed under the inability to urinate for 8 hours. At home he
fascia lata was diagnosed, lanced and widely had taken antispasmodics and had a warm bath
drained, necrotic tissues were removed. Anti- but no improvement occurred. Objectively:
biotic therapy with cephalosporines and the abdomen is soft and painful above the pubis;
III generation fluoroquinolones was prescri- dullness of percussion sound is observed above
bed, as well as immune corrective and detoxifi- the pubis. Murphy’s (Pasternatski’s) punch sign
cation infusion therapy (2,5 liters per day), di- is negative on the both sides. What condition
uresis stimulation, UV irradiation of blood and does the patient have?
plasmapheresis. What is the main component
of sepsis prevention? A. Acute urinary retention
B. Paradoxal ischuria
A. Surgical invasion C. Shronic urinary retention
B. General antibiotic therapy D. Anuria
C. Topical antibiotic therapy E. Oliguria
D. Immune correction therapy
E. Detoxification therapy 194. A 37-year-old patient has been undergoing
treatment of diskogenic lumbosacral radiculitis
190. An Rh-negative woman with 32-week- for a month. There is skin numbness observed
long term of pregnancy has been exami- at the lateral surface of the right lower extremi-
ned. It was observed that Rh-antibodies ti- ty, Achilles jerk is absent. MRI of lumbar spine
ter had increased four times within the last revealed intervertebral disk L5 -S1 prolapse up
2 weeks and was 1:64. First two pregnancies to 8 mm. Choose the further tactics for patient
ended in antenatal death of fetus caused by treatment:
hemolytic disease. What tactics of pregnancy
management should be chosen? A. Surgical extraction of intervertebral disk
B. Conservative treatment
A. Preterm delivery C. Stabilizing spinal surgery
B. Delivery at 37 weeks term D. Corset
C. Rh-antibody test in 2 weeks; if Rh-antibodies E. Manual therapy
increase in number conduct delivery
D. Introduction of anti-Rh immunoglobulin 195. A 40-year-old patient complains of
E. US examination to determine signs of fetal constant moderate pain in the lumbar spine
erythroblastosis and significantly reduced mobility. The pati-
ent has been suffering from this condition for
191. A 2-year-old child has been suffering the last 7 years since pain appeared first in the
since birth from recurring inflammatory di- sacrum area. X-ray: ankylosis of sacroiliac arti-
seases of lungs, purulent pansinusites, heari- culation, significant narrowing of intervertebral
ng deterioration, multiple cylindrical bronchi- joint fissures of lumbar vertebrae and calcifi-
ectases. Dextrocardia is observed. On biopsy: cation of spinal ligaments. What pathology is
Крок 2 Medicine (англомовний варiант, iноземнi студенти) 2015 рiк 24

most likely to cause such X-ray image? 198. A 30-year-old woman complains of
irregular copious painful menstruations, pain
A. Ankylosing spondylitis irradiates to the rectum. Anamnesis states 10-
B. Spinal tuberculosis year-long infertility. On bimanual examination:
C. Spinal osteochondrosis uterus is of normal size; uterine appendages
D. Vertebral osteochondropathy on the both sides are corded, with rectricted
E. Rheumatoid arthritis mobility, painful; there are dense nodular pai-
nful growths detected in the posterior fornix. A
196. A 7-year-old child became ill again 2 doctor suspects endometriosis. What method
weeks after he had tonsillitis. There are the allows to verify this diagnosis?
following complaints: temperature rise up to
38o C , hemorrhagic rash on the extremities, A. Laparoscopy
enlargement of the ankle joints. Blood test: B. Diagnostic curettage of uterine cavity
hemoglobin is 120 g/l, platelets are 170 · 109 /l, C. Paracentesis of posterior fornix
ESR is 30 mm/h. Urine test: proteinuria up D. Uterine probing
to 0,7 g/l, cylinders - 5-6 in the field of visi- E. Hysteroscopy
on, erythrocytes - 8-10 in the field of vision.
What mechanism of hemorrhagic syndrome is 199. A 14-year-old girl complains of tooth
present in the given case? caries; the tooth should be filled. Anamnesis
states that artificial mitral valve was installed 2
A. Vessel wall damage caused by immune years ago due to mitral insifficiency. What anti-
complexes bacterial drug should be prescribed to prevent
B. Platelet dysfunction infective endocarditis?
C. Suppression of hematopoietic stem cells
D. Decrease of adhesive-aggregative function A. Amoxicillin
of platelets B. Lincomycin
E. Vessel wall damage caused by bacteria C. Ceftriaxone
D. Erythromycin
197. A 48-year-old patient was delivered to a E. Midecamycin
hospital in-patient unit with uterine bleeding
that occurred after the 2-week-long delay of 200. A 30-year-old woman complains of inferti-
menstruation. Anamnesis states single birth. lity during her 10-year-long married life.
Examination of the uterine cervix with mirrors Menstruations occur since she was 14 and are
revealed no pathologies. On bimanual exami- irregular, with delays up to a month and longer.
nation: uterus is of normal size, painless, mobi- Body mass is excessive. Hirsutism is observed.
le; uterine appendages have no changes. Di- On bimanual examination: uterine body is
scharge is bloody and copious. What primary decreased in size; ovaries are increased in si-
hemostatic measure should be taken in the gi- ze, dense, painless, and mobile. The most likely
ven case? diagnosis is:
A. Fractional curettage of uterine cavity A. Stein–Leventhal syndrome (Polycystic ovary
B. Hormonal hemostasis syndrome)
C. Hemostatics B. Follicular cyst of ovaries
D. Uterine tamponade C. Genital endometriosis
E. Uterotonics D. Genital tuberculosis
E. Inflammatory tumor of ovaries
INSTRUCTIONAL BOOK
Testing Board

TEST ITEMS FOR LICENSING EXAMINATION: KROK 2. MEDICINE.

Kyiv. Testing Board.


(English language).

Approved to print 13.03/№63/2. Paper size 60х84 1/8


Offset paper. Typeface. Times New Roman Cyr. Offset print.
Conditional print pages 24. Accounting publishing pages 28.
Issue. 1251 copies
List of abbreviations

A/G Albumin/globulin ratio


A-ANON Alcoholics anonymous
ACT Abdominal computed tomography
ALT Alanin aminotranspherase
AP Arterial (blood) pressure
AST Aspartat aminotranspherase
BP Blood (arterial) pressure
BR Breathing rate
bpm Beats per minute
C.I. Color Index
CBC Complete blood count
CHF Chronic heart failure
CT Computer tomography
DIC Disseminated intravascular coagulation
DCC Doctoral controlling committee
DM-2 Non-Insulin dependent diabetes mellitus
DTP Anti diphtheria-tetanus vaccine
ECG Electrocardiogram
ESR Erythrocyte sedimentation rate
FC Function class
FEGDS Fibro-esphago-gastro-duodenoscopy
Gy Gray
GIT Gastrointestinal tract
Hb Hemoglobin
HbA1c Glycosylated hemoglobin
Hct, Ht Hematocrit
HDL High-density lipoproteins
HR Heart rate
IDDM Insulin dependent diabetes mellitus
IHD Ischemic heart disease
IU International unit
MSEC Medical and sanitary expert committee
NIDDM Non-Insulin dependent diabetes mellitus
pCO2 CO2 partial pressure
pO2 O2 partial pressure
Pm Per minute
Ps Pulse rate
R Roentgen
RBC Red blood count
Rh Rhesus
RR Respiratory rate
S1 (S1) Heart sound 1
S2 (S2) Heart sound 2
TU Tuberculin unit
U Unit
USI Ultrasound investigation
V/f Vision field
WBC White blood count
X-ray Roentgenogram

You might also like